SlideShare a Scribd company logo
1 of 145
Dreamz Learning Innovations_____________________________________________ Page 1
41. Migrating (wandering) acetabulum is seen in
A. Fracture dislocation of
B. Acetabulum in advanced stage of
Tuberculous hip
C. proximal femoral deficiency
D. perthes' disease
Dreamz Learning Innovations_____________________________________________ Page 2
41. Migrating (wandering) acetabulum is seen in
A. Fracture dislocation of
B. Acetabulum in advanced stage of
Tuberculous hip
C. proximal femoral deficiency
D. perthes' disease
B
Dreamz Learning Innovations_____________________________________________ Page 3
Radiological Features
To begin with there is reduction in the joint space with
juxta-artcular osteopenia. In advanced stage of arthritis
the radiological appearance of hip has been classified
by Prof TK Shanmuga sundaram into:
1. Normal type
2. Wandering acetabulum: There occurs destruction of
the acetabulum in its superior due to the disease. The
temporal head shifts proximally on the ilium. This is
called wandering acetabulum.
Dreamz Learning Innovations_____________________________________________ Page 4
3. Dislocated acetabulum.
4. Pertheoid type: The radiographic picture looks
somewhat similar to that of the Perthes disease.
5. Atrophic type.
6. Protrusio acetabuli.
Dreamz Learning Innovations_____________________________________________ Page 5
42. Radial nerve is known to be involved in which of
the following injuries
A. Fracture neck of Humerus
B. Fracture shaft of Humerus , lower third
C. Fracture Humerus distal end
D. Dislocation of elbow
Dreamz Learning Innovations_____________________________________________ Page 6
42. Radial nerve is known to be involved in which of
the following injuries
A. Fracture neck of Humerus
B. Fracture shaft of Humerus , lower third
C. Fracture Humerus distal end
D. Dislocation of elbow
B
Dreamz Learning Innovations_____________________________________________ Page 7
43. Which of the following fracture is best treated by
surgery?
A.Fracture Intertrochanteric
B.Fracture neck of femur
C.Fracture tibial condyles
D. Fracture Scaphoid
Dreamz Learning Innovations_____________________________________________ Page 8
43. Which of the following fracture is best treated by
surgery?
A.Fracture Intertrochanteric
B.Fracture neck of femur
C.Fracture tibial condyles
D. Fracture Scaphoid
B
Dreamz Learning Innovations_____________________________________________ Page 9
Femoral neck fractures usually require surgery for
treatment
Dreamz Learning Innovations_____________________________________________ Page 10
44. Osteoporosis is characterized by
A. Decreased mineralization of bone
B. Decrease in marrow content of bone
C. Decrease in bone mass
D. Softening and deformity of bone
Dreamz Learning Innovations_____________________________________________ Page 11
44. Osteoporosis is characterized by
A. Decreased mineralization of bone
B. Decrease in marrow content of bone
C. Decrease in bone mass
D. Softening and deformity of bone
C
Dreamz Learning Innovations_____________________________________________ Page 12
45. Sequestmm is a
A. Living bone formed in Osteomyelitis
B. Dead infected bone formed in Osteomyelitis
C. Bony thickening felt in Osteomyelitis
D. New bone formed in soft tissue planes
Dreamz Learning Innovations_____________________________________________ Page 13
45. Sequestmm is a
A. Living bone formed in Osteomyelitis
B. Dead infected bone formed in Osteomyelitis
C. Bony thickening felt in Osteomyelitis
D. New bone formed in soft tissue planes
B
Dreamz Learning Innovations_____________________________________________ Page 14
46. In case of Poliomyelitis, 'residual stage' of
paralysis is called after
A. 3 months of onset of acute Poliomyelitis
B. 6 months of onset of acute Poliomyelitis
C. 12 months of onset of acute Poliomyelitis
D. 24 months of onset of acute Poliomyelitis
Dreamz Learning Innovations_____________________________________________ Page 15
46. In case of Poliomyelitis, 'residual stage' of
paralysis is called after
A. 3 months of onset of acute Poliomyelitis
B. 6 months of onset of acute Poliomyelitis
C. 12 months of onset of acute Poliomyelitis
D. 24 months of onset of acute Poliomyelitis
D
Dreamz Learning Innovations_____________________________________________ Page 16
Residual Paralysis: As the acute phase of illness (0-4
weeks) subsides, the recovery begins in paralyzed
muscles. The extent of recovery is variable ranging from
mild to severe residual paresis at 60 days, depending
upon the extent of damage caused to the neurons by
the virus. Maximum neurological recovery of the
paralyzed muscle takes place in the first six months of
the illness but slow recovery continues up to two years.
After two years, no more recovery is expected and the
child is said to have “Post Polio residual paralysis”,
which remains as such through out life. However, the
child can learn to use muscles which were not
paralyzed to compensate for lost muscle power.
Dreamz Learning Innovations_____________________________________________ Page 17
47. Syme's Amputation is done through
A. Ankle joint
B. Knee joint
C. Hip joint
D. Lower third of tibia
Dreamz Learning Innovations_____________________________________________ Page 18
47. Syme's Amputation is done through
A. Ankle joint
B. Knee joint
C. Hip joint
D. Lower third of tibia
A
Dreamz Learning Innovations_____________________________________________ Page 19
Syme's Amputation
includes ankle disarticulation, removal of malleoli,
& anchoring heel pad to the wt bearing surface)
allows execellent gait with a cosmetic
prosthesis;
- symes amputations will not heal w/o palpable
posterior artery pulse;
- surgery may be performed in 2 stages;
- most common indication for this is infection;
- ankle is disarticulated in the first stage and the
amputation is revised approximately 6 weeks later
during the second stage;
Dreamz Learning Innovations_____________________________________________ Page 20
- components of the 2nd stage include resection of
the malleoli flush w/ joint surface, fixation of the fat
pad to residual bone, and revision of redundant
skin;
- this amputation allows for intermittent wt
bearing; however, skin break down may occur if a
prosthesis is used on a regular basis;
- majority of patients w/ Syme's amputation will
require prothesis designed to relieve end weight
bearing;
Dreamz Learning Innovations_____________________________________________ Page 21
Dreamz Learning Innovations_____________________________________________ Page 22
Dreamz Learning Innovations_____________________________________________ Page 23
48. Indication of open amputation is
A.Dry gangrene
B.Buerger’s gangrene
C.Gas gangrene
D.Rayrnaud's disease
Dreamz Learning Innovations_____________________________________________ Page 24
48. Indication of open amputation is
A.Dry gangrene
B.Buerger’s gangrene
C.Gas gangrene
D.Rayrnaud's disease
C
Dreamz Learning Innovations_____________________________________________ Page 25
Most surgeons agree that early and adequate surgery is
the most effective and primary means of treating
clostridial myositis. If the diagnosis is made early, while
the gangrene is relatively localized and incipient, radical
decompression of the involved fascial compartments by
free longitudinal incisions and excision of the infected
muscle usually arrest the process and eliminate the need
for amputation. Local excision preferred to amputaion,
whenever posibie, to conserve a functional extremity. If
diagnosis is reached when the process is extensive and
has caused irreversible gangrenous changes implying
permanent loss of function of the limp, open
amputation of the guillotine type or some modification
becomes necessary.
Dreamz Learning Innovations_____________________________________________ Page 26
49. Fracture of lower end of Radius is also called
A. Bennett’s fracture
B. Colles’ fracture
C. Jones’ fracture
D. Boxer’s fracture
Dreamz Learning Innovations_____________________________________________ Page 27
49. Fracture of lower end of Radius is also called
A. Bennett’s fracture
B. Colles’ fracture
C. Jones’ fracture
D. Boxer’s fracture
B
Dreamz Learning Innovations_____________________________________________ Page 28
A Jones fracture is a
fracture of the diaphysis
of the fifth metatarsal
of the foot. The fifth
metatarsal is at the
base of the small toe.
Dreamz Learning Innovations_____________________________________________ Page 29
Bennett's fracture is a fracture of the base of the
first metacarpal bone which extends into the
carpometacarpal (CMC) joint. This intra-articular
fracture
Dreamz Learning Innovations_____________________________________________ Page 30
A "Boxer's fracture" is the second and/or third
metacarpal transverse neck fracture that is more
likely to occur from a straight punch.
The "Boxer's" designation is suggestive of the
generally well tolerated way of striking a hard object
with the closed fist, with the second and third
metacarpal bones, of the index and middle fingers
respectively, absorbing most of the force.
Dreamz Learning Innovations_____________________________________________ Page 31
50. The primary site of infection in Pott’s spine is
A. Vertebral body
B. inter-vertebral disc
C. Transverse process
D. Spinous process
Dreamz Learning Innovations_____________________________________________ Page 32
50. The primary site of infection in Pott’s spine is
A. Vertebral body
B. inter-vertebral disc
C. Transverse process
D. Spinous process
B
Dreamz Learning Innovations_____________________________________________ Page 33
Pott disease (also known as tuberculous spondylitis)
refers to vertebral body and intervertebral
disc involvement with tuberculosis
Dreamz Learning Innovations_____________________________________________ Page 34
51. Most sensitive test for Anterior cruciate
ligament tear
A. Apley's grinding test
B. Drawer test
C. Lachman test
D. Me murray's test
Dreamz Learning Innovations_____________________________________________ Page 35
51. Most sensitive test for Anterior cruciate
ligament tear
A. Apley's grinding test
B. Drawer test
C. Lachman test
D. Me murray's test
C
Dreamz Learning Innovations_____________________________________________ Page 36
LACHMAN-TEST - is the most sensitive test for ACL
tear
It is done with the knee flexed at 20 degree,
stablizing the distal femur with one hand and
pulling forward on the proximal tibia with the other
hand with an intact ligament minimal translation of
the tibia occurs and a firm end point is felt with a
torn ACL more translation is noted and the end
point is soft or mushy
• The blood supply of ACL is primarily derived from
Middle genicular artery***
Dreamz Learning Innovations_____________________________________________ Page 37
ANTERIOR -DRAWER TEST - is done with the knee at
90° degree of flexion and is not as sensitive as
Lachman test but serves as an adjunct in the
evaluation of ACL instability
LOSEE TEST - THE PIVOT- SHIFT- PHENOMENON
Demonstrate the instability associated with an ACL
tear,
Dreamz Learning Innovations_____________________________________________ Page 38
'As described by losee, a valgus and internal rotation
force is applied to the tibia. Starting at 45 degree of
flexion. The lateral tibial plateau is reduced. Extending
the knee causes the lateral plateau to subluxate
anteriorly with a thud at approximately 20 degrees of
flexion. It reduces quietly at full extension.
Posterior drawer test - evaluates the integrity of the
PCL
MC- Murray test - with the Me Murray test forced
flexion and rotation of the knee elicits a clunk along
the joint line if there is meniscus injury**
Investigation of choice in Knee joint injury is MRI
Dreamz Learning Innovations_____________________________________________ Page 39
Anatomic correlation of clinical ligament instability
examination of the knee joint
Direction of force Position Ligament instability
Varus or valgus Full extension Posterior cruciate/ posterior capsule
Varus Flexion at 30 degree Lateral collateral ligament /capsule
Valgus Flexion at 30 degree Medial collateral ligament
Anterior Flexion at 30° neutral
position (AP)
Anterior cruciate ligament
Anterior Flexion at 90° neutral
internal or external
rotation
Anterior cruciate ligament
Posterior 90° (Sag test) Posterior cruciate ligament
Dreamz Learning Innovations_____________________________________________ Page 40
Important Points
• Medial meniscus has less mobility than the lateral
meniscus and is more susceptible tearing when
trapped between the femoral condyle and tibial
plateau
• Anterior cruciate ligament prevents anterior
translation and rotation of the tibia on the femur,
• Posterior cruciate ligament prevents posterior
subluxation of the tibia on the femur
• Posterior glenohumeral instability can be tested by
Jerk Test
• Coronary ligament of the knee axe —ligaments
connecting the menisci to tibia***
Dreamz Learning Innovations_____________________________________________ Page 41
52. Sudden forceful rapture of the quadriceps
femoris muscle will lead to rupture of
A. Vastus intermedius
B. Vastus lateralis
C. Rectus femoris
D. Vastus mediali's
Dreamz Learning Innovations_____________________________________________ Page 42
52. Sudden forceful rapture of the quadriceps
femoris muscle will lead to rupture of
A. Vastus intermedius
B. Vastus lateralis
C. Rectus femoris
D. Vastus mediali's
C
Dreamz Learning Innovations_____________________________________________ Page 43
Tear of Quadriceps Tendon - occurs most often in patients
over the age of 40, apparent tears that represent avulsions
from the patella occur in patients with renal osleodystrophy
or hyperparathyroidism. The tear commonly results from
sudden deceleration, such as stumbling or stipping on a wet
surface. A small flake of bone may be avulsed from the
superior pole of the patella or the tear may occur entirely
through tendinous and muscular tissues
Quadriceps rupture at the upper margin of the patella - This is
commonly known as an avulsion of the rectus femoris. It
occurs in elderly patients. It is diagnosed by the palpation of a
gap in the extensor apparatus just above the patella two
fingers can be placed in the gap usually the lateral expansions
are intact (Watson & Jones).
Dreamz Learning Innovations_____________________________________________ Page 44
Muscles Actions
• Rectus femoris • Extension of leg at knee joint flexes thigh at hip
joint***
• Vastus medialis • Extension of leg at knee joint: Stabilies patella
• Vastus intermedius • Extension of leg at knee joint: articularis genus
retracts synovial membrane
• Vastus lateralis • Extension of leg at knee joint
• Biceps femoris • Chief flexors of knee. They are weak extensors of
the hip particularly in walking
• The rectus femoris muscle can rupture in sudden violent extension
movement of the knee joints**
• Vastus medialis is the first part of the quadriceps muscle to atrophy in
knee joint and the last to recover***
Dreamz Learning Innovations_____________________________________________ Page 45
53. Von- rosen splint are applicable in
A. Developmental dysplasia of hip joint
B. Congenital talipes equinovarus
C. Congenital vertical talus
D. Tibia mileruae
Dreamz Learning Innovations_____________________________________________ Page 46
53. Von- rosen splint are applicable in
A. Developmental dysplasia of hip joint
B. Congenital talipes equinovarus
C. Congenital vertical talus
D. Tibia mileruae
A
Dreamz Learning Innovations_____________________________________________ Page 47
VON-ROSEN'S SPLINT - is an H-shaped melleable
splint
• Object of splintage is to hold the hips some what
flexed and abducted
• Used in congenital dislocation / dysplasia of hip
joint
Also - Remember
• Frog leg or Lorenz cast or Batchelor cast used in
CDH
• Broomstick plaster used in perthe's disease
• Patella Tendon Bearing (PTB) cast used in- Both
bones leg fracture
Dreamz Learning Innovations_____________________________________________ Page 48
• Gutter splint- phalyngeal and metaphalyngeal
fractures
• Denis Browne splint used in CTEV, Stirrup splint
Ankle fractures
• Knuckle-bender splint- ulner nerve palsy, Cock up
splint - Radial nerve palsy
• Milwaukee brace (Cervical thoracolombosacral
orthosis) used in Scoliosis
Dreamz Learning Innovations_____________________________________________ Page 49
• Risser's localizer cast, Turn buckle cast- Scoliosis
• Minerva cast - cerical spine disease
• U slab or Hanging cast - fracture of humerus
• Tube cast (Cylindrical cast) - used for fracture around
knee (# patella)
• Halo cast (vest) - for spinal injury
• Velpeau bandage and sling and Swathe splint are
used in - Shoulder dislocation**
• Thromboembolic disease (Pulmonary embolism) is
the most common cause of death after Total hip
replacement*** (Watson jones operation)
Dreamz Learning Innovations_____________________________________________ Page 50
• Pectorafis major and minor muscles are the most
common congenitically absent muscles in
humans**
• Pollicization refer to reconstruction of the thumb
with a substitute finger (i.e. index finger)
• Bohler's Tuber Joint angle and crucial angle of
Gissane are measured for intraarticular #
ofcalcanium
Development Dysplasia of the hep (DDH)
• More common in female, short stature**
• Oligohydramnios is associated with a higher risk of
DDH
Dreamz Learning Innovations_____________________________________________ Page 51
• The hourglass appearance of the capsule may
prevent a successful closed reduction
• In a child with bilateral dislocation - Waddling gait-
unilateral dislocation Trendelenburg's gait*
• X-ray shows break in shenton's line*
• Striking feature is widening of perineum & marked
lumber lorodosis*
• Wandering acetabulum is sen in T.B of hip joints
Dreamz Learning Innovations_____________________________________________ Page 52
Angles of Importance
(i) Cobb's angle - Scoliosis
(ii) Kite's angle - CTEV
(iii) Meary's angle - Pes cavus
(iv) Hilagenreiner's epiphyseal angle - congenital
coxa-vera
(v) Baumann's angle - Supracondylar #
Rocker bottom foot or vertical talus is associated
with Arthrogrypasis multiplex congenital spina
bifida & trisomy 13-18
Dreamz Learning Innovations_____________________________________________ Page 53
54. Congenital dislocation of hip is common in
A. Chinese
B. Asian
C. Negro
D. North American Indians
Dreamz Learning Innovations_____________________________________________ Page 54
54. Congenital dislocation of hip is common in
A. Chinese
B. Asian
C. Negro
D. North American Indians
D
Dreamz Learning Innovations_____________________________________________ Page 55
CONGENITAL - DISLOCATION OF THE HIP (CDH) - In
western races. It is one of the commonest congenital
disorder but it is uncommon in India and some other Asian
countries probably because of the culture of mother
carrying the child on the side of their waist with the hips
of the child abducted. This position helps in the reduction
of an unstable hip which otherwise would have dislocated
(200-M)
• Dislocation is very common in Lapps and North
American Indians who swaddle their babies and carry
them with legs together, hips and knees fully extended
and is rare in southern Chinese and African Negroes who
carry their babies astride their backs with legs widely
abducted (Apley)
Dreamz Learning Innovations_____________________________________________ Page 56
• Investigation of choice in newborn is real time
ultrasound
Aetiology and pathogencsis
• Genetic factor - dominant trait, and acetabular
dysplasia
• Hormonal factors - High levels of maternal
oestrogen progesterone and relaxin
• Breach position with extended legs
• Postnatal factors - contribute to persistence of
neonatal instability and acetabular maldevelopment
Dreamz Learning Innovations_____________________________________________ Page 57
Diagnosis of CDH
• Limitation of the abduction
of the hip
• Ortoloni'test is positive (Jerk of
entry')
• Asymmetrical thigh fold • Barlow's test
• High buttock fold on the
affected side
• Telescopy positive
• Galeazzi 's sign • Trendelenburg, waddling gait*
• Limb is short and slightly
externally rotated
• Trendelenburg's test
positive
Dreamz Learning Innovations_____________________________________________ Page 58
Important X-ray findings in CDH
- Delayed appearance of ossific centre for the head of
the femur .
- Retarded development of ossific centre of the head of
the femur
- Slopping acetabulum
- Lateral and upward displacement of the ossific centre
of the femoral head
- A break in shenton 's line***
Dreamz Learning Innovations_____________________________________________ Page 59
55. Most common organism in drug abusers chronic
osteomyelitis is
A. Staph.aureus
B. Pseudomonas
C. Granulmatous
D. Klebsiella
Dreamz Learning Innovations_____________________________________________ Page 60
55. Most common organism in drug abusers chronic
osteomyelitis is
A. Staph.aureus
B. Pseudomonas
C. Granulmatous
D. Klebsiella
B
Dreamz Learning Innovations_____________________________________________ Page 61
Most common organisms in Osteomyelitis
• Acute osteomyelitis- children under 4 years of age. H.
influenme
• Acute osteomyelitis in older children, adults - Staph.aureus
• Sickle cell disease - salmonella
• Heroin addicts and immunocopromised—Pseudomonas,
aeniginosa, proteus mirabilis, Bacteroids
• Sub acute haematogenous osteomyelitis - Staph. aureus
• Post traumatic osteomyelitis, Infected metal implants
Staph. aureus
Dreamz Learning Innovations_____________________________________________ Page 62
• Post operative osteomyelitis - Mixed infection (Staph.
aureus Proteus, Pseudomonas)
• Osteomyelitis of the spine - Staph, aureus, Pseudomonas
aeruginosa
• Squamous cell carcinoma arising from a chronic
osteomyelitis - occurs in areas of burn scars, chronic pressure
ulcers, and ostomies as well as as sites of chronic draining
osteomyelitis Stable chronic osteomyelitis for an average of
approximately 20 years
Dreamz Learning Innovations_____________________________________________ Page 63
Earliest site of bone involvement in hematogenous
osteomyelitis is - Metaphysis**
Markers of Bone formation Markers of Bone resorption
• Serum bone specific alkaline
phosphatase**
• Serum osteocalcin
• Serum propeptide or type I
procollagen
• Urine and serum N- telopeptide
• Urine and serum C- telopeptide*
• Urine total free deoxypyridinoline
• Urine Hydroxyproline*
• Serum tartarate resistant- Acid
phosphatase*
• Serum bone sialoprtein
• Urine hydroxylysine glycosides
Dreamz Learning Innovations_____________________________________________ Page 64
• Infective endocarditis due to pseudomonas is most
commonly seen with IV drug abuse of pentazocin**
Classification systems for Osteomyelitis
1. Traditional system - Acute (<2 weeks) subacute (weeks to
months) chronic (>3months)
2. Waldvogel system
- Hematogenous
- Arising from contiguous infection No vascular disease
- Vascular disease present chronic
3. Cierny- Mader system
Anatomic extent of infection
1. Medullary only (acute hematogenous)
2. Superficial cortex (continguous spread or soft tissue
trauma)
Dreamz Learning Innovations_____________________________________________ Page 65
3. Localized (cortical and medullary, mechanically stable)
4. Diffuse (cortical and medullary mechanically unstable)
5. Subtype by host's physiologic status
A Healty
Bs Compromised because of systemic factors
B1 Compromised because of local factors
B1s Compromised because of both local and systemic
factors
C Treatment worse than the disease
Dreamz Learning Innovations_____________________________________________ Page 66
56. A-man is stucked with lathi,below the knee in
lateral aspect of the leg. He drags the toes and
cannot feel the sensation in dorsum of feet which of
the following is intact
A. Inversion
B. Eversion
C. Dorsiflexion
D. Loss of sensation in 1st & 2nd toes
Dreamz Learning Innovations_____________________________________________ Page 67
56. A-man is stucked with lathi,below the knee in
lateral aspect of the leg. He drags the toes and
cannot feel the sensation in dorsum of feet which of
the following is intact
A. Inversion
B. Eversion
C. Dorsiflexion
D. Loss of sensation in 1st & 2nd toes
A
Dreamz Learning Innovations_____________________________________________ Page 68
Clinical picture suggest diagnosis is a lateral
popliteal nerve injury
• Causes of Lateral popliteal nerve palsy (Waston &
Jones')
• Rupture of the external lateral ligament
• Avulsion of the styloid process of the fibula
• Dislocation of the knee joint
• Compressed by. Splint, POP or strapping
• Ganglion arising from superior tibio-fibular joint
• Cyst of the lateral meniscus
Dreamz Learning Innovations_____________________________________________ Page 69
Clinical features
• Paralysis of the anterior tibial and peroneal
muscle
• Loss of sensibility in the outer aspect of the leg
and dorsum of the foot
PERONEAL NERVE (Apley)
• Injuries may affect either the common peroneal
{lateralpopliteal) nerve or one of its branches the
deep or superficial peroneal nerve
Dreamz Learning Innovations_____________________________________________ Page 70
[A]. Common peroneal nerve (Lateral popliteal
nerve)
• The patient has drop foot and can neither
dorsifiex nor evert the foot**
• He walks with a high stepping gait to avoid
catching the toes
• Sensation is lost over the front and outer half of
the leg and the dorsum of the foot**
• Planter flexion & Inversion are normal**
Dreamz Learning Innovations_____________________________________________ Page 71
[B]. Deep peroneal nerve
• Pain and weakness of dorsiflexion
• Sensory loss in a small area of skin between the first and second
toes**
• Paraesthesia and numbness on the dorsum around the first web
space
[C]. Superficial peroneal nerve
• Pain in the lateral part of the leg and numbness or paraesthesia
of the foot
• There may be weakness of eversion and sensory loss on the
dorsum of the foot
• Most commonly injured in misplaced gluteal injection - Sciatic
nerve injury
• Most common cause of insertional tendonitis of tendoachilis is -
over use**
Dreamz Learning Innovations_____________________________________________ Page 72
57. True statement regarding posterior dislocation
of shoulder joint
A. External rotation
B. Internal rotation
C. Axillary nerve palsy
D. Anterior hollowness
Dreamz Learning Innovations_____________________________________________ Page 73
57. True statement regarding posterior dislocation
of shoulder joint
A. External rotation
B. Internal rotation
C. Axillary nerve palsy
D. Anterior hollowness
B
Dreamz Learning Innovations_____________________________________________ Page 74
• Anterior dislocation is much more common than
posterior
Posterior dislocation of Schoulder is rare
Mechanism of injury — Indirect force producing
marked internal rotation* and adduction* must
needs be very sever to cause a dislocation. This
happens most commonly during a fit or convulsions
or with electric shock.
Dreamz Learning Innovations_____________________________________________ Page 75
ANTERIOR SHOULDER DlSLOCATION POSTERIOR SHOULDER
DISLOCATION
• Due to shallowness of the glenoid socket, the
extra ordinary range of the movement,
ligamentous laxity or glenoid dysplasia
• The lateral out line of the shoulder may be
flattened, bulge may be felt just below the
clavicle
• The front of the shoulder looks flat
with a prominent coracoid
• Posterior displacement is usually
apparent
• The characteristic physical sign is loss
of lateral rotation of the shoulder which
is locked in medial rotation (Watson &
Jones)
• Posterior dislocations are often
difficult to diagnose because the patient
may have a normal contour to the
shoulder or the deltoid of a well
developed athlete may mask signs of a
displaced humeral head (207-current
diagnosis & treatment 4th)
• A "reverse Hill- Sachs lesion" may
appear on the anterior articular surface
of the humerus
Patho physiological changes
• Bankart's lesion — Dislocation causes stripping
of the glenoidal labrum along with the
periosteum from the anterior surface of the
glenoid and scapular neck***
• Hill- Sach 's lesion — is a depression on the
humeral head in its postero-lateral quadrant.
• Eburnated rim of the glenoid
• Associated # of the GT of the humerus or rim of
the glenoid
Dreamz Learning Innovations_____________________________________________ Page 76
• Dugas' test, Hamilton ruler test, callway's test are
positive in Anterior dislocation of the shoulder joint
• Axillary nerve and Axillary artery injury is common in
Anterior dislocation of the shoulder joint
Empty glenoid sign - in AP- film the humeral head
because it is medially rotated, looked abnormal in
shape (like an electric light bulb) and it stands away
somewhat from glenoid fossa is seen in posterior
dislocation of the shoulder joint
• Patella almost always dislocates laterally
• Recurrent dislocations are extremely rare in the
Ankle joints*
Dreamz Learning Innovations_____________________________________________ Page 77
Recurrent dislocations of shoulder joints
Three important lesions
1. Hill sach's lesions**
2. Bankart's lesion**
3. Erosion of anterior rim
of glenoid cavity
Oilier pathologic lesions
1. Capsular laxity*
2. Sub capsular's deficiency
3. Glenoid fossa deficiency
Dreamz Learning Innovations_____________________________________________ Page 78
Anterior cruciate ligament Posterior cruciate ligament
• Prevent anterior translation of the
tibia on the femur**.limits hyper
extension
• Attached on tibia just behind
anterior horn of medial meniscus
Runs upwards bat Km aids and
laterally
• Attached on femur over medial
surface of lateral femoral condyle
• It is intracapsular and
intrasynoviai*
• It is tout during extension of knee
• Prevent posterior translation on
tibia on the femur**
• Attached on tibia behind posterior
horn of medialmeniscus Runs
upwards forwards and medially
• Attached on femur over lateral
surface of medial femoral condyle
• It is intracapsular but
extrasynovial*
• It is tout during flexion of knee*
Dreamz Learning Innovations_____________________________________________ Page 79
58. Following are reduce the infection rate in
implant surgery EXCEPT
A. Laminar flow
B. Low air flow
C. Body exaust custom
D. Frequent hand washing
Dreamz Learning Innovations_____________________________________________ Page 80
58. Following are reduce the infection rate in
implant surgery EXCEPT
A. Laminar flow
B. Low air flow
C. Body exaust custom
D. Frequent hand washing
B
Dreamz Learning Innovations_____________________________________________ Page 81
Cohart placement in an area with an independent air
supply, exhaust system & bathroom facilities
• Surgeons may use double gloves, space suits adherent
plastic drapes, pulstile lavage, and laminar flow operating
rooms or rooms with ultraviolet lights as additional
techniques that may lower the risk of intraoperative
contamination
The main preventive measures of Infection control are
1. Isolation - Infectious patients must be isolated
2. Hospital staff- Those who are suffering from skin
diseases, sore throat, common cold, ear infection diarrhea
or dysentery and other infectious ailments should be kept
away from work until completely cured
Dreamz Learning Innovations_____________________________________________ Page 82
3. Hand washing - The most common route of
infection is via the hands. Hand washing with soap
and water may not be sufficient, a suitable
disinfectant must be employed for handwashing
4. Dust control - Hospital dust contains numerous
bacteria and viruses, suppression of dust by wet
dusting and vaccume cleaning are important control
measures
5. Disinfection - The article used by the patient as
well as patients urine, faeces, sputum should be
properly disinfected proper sterilization of
instrument should be enforced
Dreamz Learning Innovations_____________________________________________ Page 83
6. Control of droplet infections-use of face masks,
proper bed spacing, prevention of over crowding
and ensuring adequate lighting and ventilation are
important control measures
7. Nurshing technique - Barrier nurshing and task
nurshing have been recommended to minimum
cross infections
8. Administrative measures - There should be a
hospital "control of infection committee" to
formulate policies regarding control of hospital
acquired infections
Dreamz Learning Innovations_____________________________________________ Page 84
COHORT NURSHING - refer to the grouping of patients with a
given infection within an isolated area short of strict (single room
isolation) cohort grouping of infectious patients & nurshing them
within an area of a hospital ward is widely recommended as a
strategy for controlling transmission of health care acquired
infections
According to WHO highly aerosolized infected cases should be
isolated & accommodated as follows in depending order or
preference
- Negative pressure room with the door closed**
- Single room with their own bathroom facilities
Cohart placement in an area with an independent air supply
exhaust system & bathroom facilities
The body exhaust system in total hip arthroplasty.
Dreamz Learning Innovations_____________________________________________ Page 85
59. Most common nerve involvement in
supracondylar fracture of humerus is
A. Median nerve
B. Ulnar nerve
C. Posterior interosseus nerve
D. Anterior interosseus nerve
Dreamz Learning Innovations_____________________________________________ Page 86
59. Most common nerve involvement in
supracondylar fracture of humerus is
A. Median nerve
B. Ulnar nerve
C. Posterior interosseus nerve
D. Anterior interosseus nerve
A
Dreamz Learning Innovations_____________________________________________ Page 87
SUPRACONDYLAR FRACTURE HUMERUS
• Radial nerve is the most commonly injured nerve,
the median nerves also sometimes affected
• Median nerve may be injured but loss of function
is usually temporary.
• The nerve most commonly affected is median :
radial palsies are rare, and the ulnar nerve is only
involved when the fracture displacement is anterior
(Watson & Jones)
• In single choice best answer is - Median nerve
(anterior interroseous branch)
Dreamz Learning Innovations_____________________________________________ Page 88
Compression of median nerve within the carpal
tunnel is the most common upper extremity
compressive neuropathy
Causes of carpal tunnel syndrome - Pregnancy
amyloidosis, flexor tenosynovitis, over use
phenomenon acute or chronic inflammatory
conditions, traumatic disorders of the wrist, DM and
hypothyroidism
• Dunlop traction is used for supracondylar fracture
of the humerus
• Best prognosis after nerve repair is - Radial**
Dreamz Learning Innovations_____________________________________________ Page 89
• Most common nerve used for monitoring during
anaesthesia is ulnar nerve
• Earliest reversal is orbicularis occuli supplied by facial
nerve
• Three bony point relationship is maintained in- supra
condylar # humerus
• Three bony point relationship is reversed m
dislocation of elbow joint
• Allen's test - is performed to identify patency of the
radial and ulnar arteries at the wrist and is indicator of
the integrity of the palmar arch, .
• Neuronal degeneration is seen in crush nerve injury
fetal development and senescence
Dreamz Learning Innovations_____________________________________________ Page 90
Condition Features & Test
Medial nerve palsy • Pointing index
• Pen test (test Abductor pollicis brevis)
• Benediction test
• Oschner clasp test & opposition loss
• Ape thumb deformity
• Carpal tunnel syndrome
Ulnar nerve • Book test (format sign)
• Card test
• Egawastest
• Musician nerve
Radial nerve • Wrist drop*
• Thumb drop*
• Finger drop*
Sciatic nerve is the thickest* nerve of the body. It is the terminal branch of the
lumbo-sacral plexus
Dreamz Learning Innovations_____________________________________________ Page 91
60. Radio-isotopes used in all EXCEPT
A. Fracture healing
B. Infections of the joints
C. Skeletal metastasis
D. Bone tumors
Dreamz Learning Innovations_____________________________________________ Page 92
60. Radio-isotopes used in all EXCEPT
A. Fracture healing
B. Infections of the joints
C. Skeletal metastasis
D. Bone tumors
A
Dreamz Learning Innovations_____________________________________________ Page 93
Clinical Applications of Radio-isotopes
1. Diagnosis of stress fracture (or other undisplaced fracture)
2. Detection of a small bone abscess or an osteoid osteoma
3. Investigation of loosening or infection around prosthesis
4. Diagnosis of femoral head ischaemia in perthe’s disease or
avascular necrosis in adults
5. Early detection of bone metastases
**the best indication for a bone scan is suspected multiple bony
lesions such as in metastatic carcinomas, and lymphoma of bone
• Bone- infarcts are seen in – Sickle cell anemia**
Dreamz Learning Innovations_____________________________________________ Page 94
61. A lateral blow at the level of the knee
joint may cause all except :
A. Rupture of anterior cruciate ligament
B. Rupture of medial collateral ligament
C. Avulsion of lateral meniscus
D. Bumper fracture of tibia
Dreamz Learning Innovations_____________________________________________ Page 95
61. A lateral blow at the level of the knee
joint may cause all except :
A. Rupture of anterior cruciate ligament
B. Rupture of medial collateral ligament
C. Avulsion of lateral meniscus
D. Bumper fracture of tibia
C
Dreamz Learning Innovations_____________________________________________ Page 96
• Avulsion of medial meniscus
Dreamz Learning Innovations_____________________________________________ Page 97
62. Osteoporosis is a deficiency in:
A. Calcium metabolism
B. Calcium deposition
C. Protein supporting tissue
D. All of the above
Dreamz Learning Innovations_____________________________________________ Page 98
62. Osteoporosis is a deficiency in:
A. Calcium metabolism
B. Calcium deposition
C. Protein supporting tissue
D. All of the above
C
Dreamz Learning Innovations_____________________________________________ Page 99
63. Achondroplasia is characterized by the
following features except:
A. Short extremities
B. Normal trunk length
C. Normal intelligence
D. Normal ossification of cartilage
Dreamz Learning Innovations_____________________________________________ Page 100
63. Achondroplasia is characterized by the
following features except:
A. Short extremities
B. Normal trunk length
C. Normal intelligence
D. Normal ossification of cartilage
D
Dreamz Learning Innovations_____________________________________________ Page 101
64. Osteogenesis imperfecta is
characterized by the following features
except:
A. Blue sclerae
B. Brittle shell-like bones
C. Multiple fractures
D. Osteoporosis
Dreamz Learning Innovations_____________________________________________ Page 102
64. Osteogenesis imperfecta is
characterized by the following features
except:
A. Blue sclerae
B. Brittle shell-like bones
C. Multiple fractures
D. Osteoporosis
D
Dreamz Learning Innovations_____________________________________________ Page 103
65. The treatment of osteoclastoma
includes the following measures except:
A. Curettage of tumor tissue & packing
cavity with bone chips
B. Excision with safety margin of bone
C. Radiotherapy
D. Chemotherapy
Dreamz Learning Innovations_____________________________________________ Page 104
65. The treatment of osteoclastoma
includes the following measures except:
A. Curettage of tumor tissue & packing
cavity with bone chips
B. Excision with safety margin of bone
C. Radiotherapy
D. Chemotherapy
D
Dreamz Learning Innovations_____________________________________________ Page 105
66. The differential diagnosis of osteoid
osteoma includes all the following
except:
A. Brodies' abscess
B. Ossifying fibroma
C. Ewing's tumor
D. Metastatic thyroid nodule
Dreamz Learning Innovations_____________________________________________ Page 106
66. The differential diagnosis of osteoid
osteoma includes all the following
except:
A. Brodies' abscess
B. Ossifying fibroma
C. Ewing's tumor
D. Metastatic thyroid nodule
D
Dreamz Learning Innovations_____________________________________________ Page 107
67. The management of Perthe's disease
includes all except
A. Broomstick plasters
B. Compression nail plating
C. Femoral osteotomy
D. Innominate osteotomy
Dreamz Learning Innovations_____________________________________________ Page 108
67. The management of Perthe's disease
includes all except
A. Broomstick plasters
B. Compression nail plating
C. Femoral osteotomy
D. Innominate osteotomy
B
Dreamz Learning Innovations_____________________________________________ Page 109
68. Common complications of plastering of
fractures is :
A. Ischemia of the limb distal to the plaster.
B. Venous congestion and venous
thrombosis.
C. Delayed or malunion of fracture.
D. Orthostatic oedema of the limb:
Dreamz Learning Innovations_____________________________________________ Page 110
68. Common complications of plastering of
fractures is :
A. Ischemia of the limb distal to the plaster.
B. Venous congestion and venous
thrombosis.
C. Delayed or malunion of fracture.
D. Orthostatic oedema of the limb:
D
Dreamz Learning Innovations_____________________________________________ Page 111
69. Treatment of Intracapsular fracture neck
of femur in adult
A. Skeletal traction
B. POP Spica
C. Excision of head of femur
D. Closed reduction and internal fixation
Dreamz Learning Innovations_____________________________________________ Page 112
69. Treatment of Intracapsular fracture neck
of femur in adult
A. Skeletal traction
B. POP Spica
C. Excision of head of femur
D. Closed reduction and internal fixation
C
Dreamz Learning Innovations_____________________________________________ Page 113
• Treatments for intracapsular femoral
neck fractures vary from internal fixation
to hemiarthroplasty to total hip
replacement
Dreamz Learning Innovations_____________________________________________ Page 114
70. Common general complications of
severe fracture pelvis include all except
A. paralytic ileus.
B. deep vein thrombosis of lower limbs.
C. crush injury.
D. pulmonary shock.
Dreamz Learning Innovations_____________________________________________ Page 115
70. Common general complications of
severe fracture pelvis include all except
A. paralytic ileus.
B. deep vein thrombosis of lower limbs.
C. crush injury.
D. pulmonary shock.
C
Dreamz Learning Innovations_____________________________________________ Page 116
71. Supracondylar fractures of the humerus in
children
A. Are common in children below the age of 10
B. More often result in a flexion injury than an
extension injury
C. May be complicated by a compartment
syndrome in the upper arm, if the brachial
artery is affected
D. Feeble radial pulse is an absolute indication
for immediate open exploration
Dreamz Learning Innovations_____________________________________________ Page 117
71. Supracondylar fractures of the humerus in
children
A. Are common in children below the age of 10
B. More often result in a flexion injury than an
extension injury
C. May be complicated by a compartment
syndrome in the upper arm, if the brachial
artery is affected
D. Feeble radial pulse is an absolute indication
for immediate open exploration
Answer A
Dreamz Learning Innovations_____________________________________________ Page 118
These fractures are common in young children.
Most are hypertension injuries. A brachial
artery compromise may cause a forearm (not
upper arm) compartment syndrome, and call
for a trial of immediate closed reduction
Dreamz Learning Innovations_____________________________________________ Page 119
72. Ulnar nerve compression: all of the following
are true, EXCEPT:
A. It may present with weakness rather than
pain
B. It usually requires nerve conduction studies
for diagnosis of site compression
C. Surgery is indicated for pain, but not for
weakness
D. it is less common than the carpal tunnel
syndrome
Dreamz Learning Innovations_____________________________________________ Page 120
72. Ulnar nerve compression: all of the following
are true, EXCEPT:
A. It may present with weakness rather than
pain
B. It usually requires nerve conduction studies
for diagnosis of site compression
C. Surgery is indicated for pain, but not for
weakness
D. it is less common than the carpal tunnel
syndrome
Answer C
Dreamz Learning Innovations_____________________________________________ Page 121
Surgery should be done if weakness is present,
even in the ab-, sence of pain.
Dreamz Learning Innovations_____________________________________________ Page 122
73. A 10-year-old boy presents with a swelling in
the upper humerus. Which of the following is
the most cause?
A. Osteosarcoma
B. Metastatic tumor
C. Myeloma
D. Lymphoma
Dreamz Learning Innovations_____________________________________________ Page 123
73. A 10-year-old boy presents with a swelling in
the upper humerus. Which of the following is
the most cause?
A. Osteosarcoma
B. Metastatic tumor
C. Myeloma
D. Lymphoma
Answer A
Dreamz Learning Innovations_____________________________________________ Page 124
Acute osteomyelitis, osteosarcomas and
Ewing's tumors are among the commonest
causes of swellings in the upper humerus in
children.
Dreamz Learning Innovations_____________________________________________ Page 125
74. Shoulder dislocation: the following are all
true EXCEPT:
A. It is the commonest of joint dislocations in
adults
B. It is predisposed to by a Hill-Sach's lesion
C. It is usually posterior
D. It can be reduced by the Kocher method
Dreamz Learning Innovations_____________________________________________ Page 126
74. Shoulder dislocation: the following are all
true EXCEPT:
A. It is the commonest of joint dislocations in
adults
B. It is predisposed to by a Hill-Sach's lesion
C. It is usually posterior
D. It can be reduced by the Kocher method
Answer C
Dreamz Learning Innovations_____________________________________________ Page 127
Most shoulder dislocations are anterior.
Dreamz Learning Innovations_____________________________________________ Page 128
75.Fractures of the proximal humerus
A. Are increased in incidence in older persons
B. Are more likely to need surgery in elderly
patients
C. Are classified according to mason's
classification
D. All of the above are true
Dreamz Learning Innovations_____________________________________________ Page 129
75.Fractures of the proximal humerus
A. Are increased in incidence in older persons
B. Are more likely to need surgery in elderly
patients
C. Are classified according to mason's
classification
D. All of the above are true
Answer A
Dreamz Learning Innovations_____________________________________________ Page 130
Humeral fractures are commoner in older
persons.
Dreamz Learning Innovations_____________________________________________ Page 131
76.A 25 year old male presents to the emergency
department after a road traffic accident. Clinical
examination shows a blood pressure of 100/70 and
mild abdominal tenderness. Iliac crest "springing" for
a pelvic fracture is positive. Which of the following is
the LEAST important next step in management?
A. Start immediate intravenous fluids
B. Conduct a careful examination for other injuries
C. Carry out immediate diagnostic peritoneal lavage
(DPL) for intra-abdominal trauma
D. Send blood for hemoglobin levels and cross-matching
Dreamz Learning Innovations_____________________________________________ Page 132
76.A 25 year old male presents to the emergency
department after a road traffic accident. Clinical
examination shows a blood pressure of 100/70 and
mild abdominal tenderness. Iliac crest "springing" for
a pelvic fracture is positive. Which of the following is
the LEAST important next step in management?
A. Start immediate intravenous fluids
B. Conduct a careful examination for other injuries
C. Carry out immediate diagnostic peritoneal lavage
(DPL) for intra-abdominal trauma
D. Send blood for hemoglobin levels and cross-matching
Answer C
Dreamz Learning Innovations_____________________________________________ Page 133
All patients with potentially significant trauma
require at least one, and preferably two, i.v.
lines for fluid infusion and blood. Every
patient requires a quick clinical examination
of the entire body. Except in the direst
circumstances, every patient requires
detailed clinical examination. Blood
transfusion in this patient may be required,
even if the hemoglobin is normal, but a DPL
is not indicated at this stage, when the
hemodynamic stability of the patient has not
been confirmed.
Dreamz Learning Innovations_____________________________________________ Page 134
77.A 55-year-old laborer presents to the casualty of
a primary health center after a fall from a height.
Plain X-rays confirm a fracture of both pubic
rami. The patient is hemodynamically stable,
but during observation of 5 hours it is noted that
he does not pass urine. At that time abdominal
examination shows that the bladder is full. If a
urologist is unavailable, the most appropriate
next step is:
A. Administer intravenous carbachol to stimulate
bladder contraction
B. Administer sympathomimetics to decrease
prostatic congestion
C. Pass a urethral catheter
D. Carry out a suprapubic catheterization ' "
Dreamz Learning Innovations_____________________________________________ Page 135
77.A 55-year-old laborer presents to the casualty of a primary
health center after a fall from a height. Plain X-rays
confirm a fracture of both pubic rami. The patient is
hemodynamically stable, but during observation of 5
hours it is noted that he does not pass urine. At that time
abdominal examination shows that the bladder is full. If a
urologist is unavailable, the most appropriate next step
is:
A. Administer intravenous carbachol to stimulate bladder
contraction
B. Administer sympathomimetics to decrease prostatic
congestion
C. Pass a urethral catheter
D. Carry out a suprapubic catheterization ' "
Answer D
Dreamz Learning Innovations_____________________________________________ Page 136
The other three steps mentioned are all
disastrous. Even in tertiary care centers,
where retrograde urethrography facilities
exist, suprapubic catheterization is never a
wrong step in a patient with pelvic fracture
and retention
Dreamz Learning Innovations_____________________________________________ Page 137
78.A 62 year old woman presents with a history of
fall while bathing. On examination she is
hemodynamically stable, but is unable to move
the left leg, which is lying in an attitude of
external rotation. The following are all true
EXCEPT:
A. The patient is likely to have an intracapsular
fractureof the femoral neck
B. Plain X-rays are enough for diagnosis, and CT is
rarely required
C. Surgical reduction and fixation or replacement is
always needed, and conservative treatment is
contraindicated
D. The commonest complication of this fracture is
malunion
Dreamz Learning Innovations_____________________________________________ Page 138
78.A 62 year old woman presents with a history of fall while
bathing. On examination she is hemodynamically stable,
but is unable to move the left leg, which is lying in an
attitude of external rotation. The following are all true
EXCEPT:
A. The patient is likely to have an intracapsular fractureof
the femoral neck
B. Plain X-rays are enough for diagnosis, and CT is rarely
required
C. Surgical reduction and fixation or replacement is always
needed, and conservative treatment is contraindicated
D. The commonest complication of this fracture is malunion
Answer D
Dreamz Learning Innovations_____________________________________________ Page 139
The commonest complications of intracapsular
fracture are nonunion and avascular
necrosis. Unlike pelvic fractures, plain X-rays
are usually enough to diagnose femoral
fractures. If the X-rays are inconclusive
despite compelling clinical signs, the patient
should undergo repeat X-rays after 2-3 days,
or MRI.
Dreamz Learning Innovations_____________________________________________ Page 140
79.A 13 year old boy sustains a closed, stable
fracture of the upper tibia. After reduction
and application of a cast, he complains of
excessive calf pain. The following step
should be taken:
A. X-rays to check for cast fragments V
B. Elevate the limb 5-10°
C. Elevate the limb by about 90°
D. Remove the cast
Dreamz Learning Innovations_____________________________________________ Page 141
79.A 13 year old boy sustains a closed, stable
fracture of the upper tibia. After reduction
and application of a cast, he complains of
excessive calf pain. The following step
should be taken:
A. X-rays to check for cast fragments V
B. Elevate the limb 5-10°
C. Elevate the limb by about 90°
D. Remove the cast
Answer D
Dreamz Learning Innovations_____________________________________________ Page 142
The boy has probably developed an impending
compartment syndrome in the leg. This is
caused by insufficient vascularity in the
affected limb. The first step is to remove the
cast and elevate the limb, and observe. If the
pain continues the patient will require a
fasciotomy.
Dreamz Learning Innovations_____________________________________________ Page 143
80. The following is true of ankle fractures:
A. The most frequent mechanism is pronation
of the forefoot and internal rotation at the
ankle
B. Swelling and tenderness are typically absent
or negligible
C. Undisplaced fractures should be treated with
a cast
D. Post-traumatic arthritis is rare at the ankle
Dreamz Learning Innovations_____________________________________________ Page 144
80. The following is true of ankle fractures:
A. The most frequent mechanism is pronation
of the forefoot and internal rotation at the
ankle
B. Swelling and tenderness are typically absent
or negligible
C. Undisplaced fractures should be treated with
a cast
D. Post-traumatic arthritis is rare at the ankle
Answer C
Dreamz Learning Innovations_____________________________________________ Page 145
Fractures at the ankle result from a twist. The
injury develops during supination of the
forefoot with external rotation at the ankle.
Tenderness is marked, and swelling occurs.
The commonest late complication is arthritis.

More Related Content

Similar to Orthopedics Class Day 7.ppt

Lecture 3 maxillofacial trauma part 3
Lecture 3 maxillofacial trauma part 3Lecture 3 maxillofacial trauma part 3
Lecture 3 maxillofacial trauma part 3Lama K Banna
 
Topic 60 CERVICAL RIB.ppt
Topic 60 CERVICAL RIB.pptTopic 60 CERVICAL RIB.ppt
Topic 60 CERVICAL RIB.pptK Ravi Kiran
 
Maxillofacial prosthesis
Maxillofacial prosthesisMaxillofacial prosthesis
Maxillofacial prosthesisUE
 
Sp06 starfish parts 1
Sp06 starfish parts 1Sp06 starfish parts 1
Sp06 starfish parts 1andrewmay123
 
Principles_of_fractures-1.ppt
Principles_of_fractures-1.pptPrinciples_of_fractures-1.ppt
Principles_of_fractures-1.pptAhmedMufleh1
 

Similar to Orthopedics Class Day 7.ppt (7)

Lecture 3 maxillofacial trauma part 3
Lecture 3 maxillofacial trauma part 3Lecture 3 maxillofacial trauma part 3
Lecture 3 maxillofacial trauma part 3
 
Topic 60 CERVICAL RIB.ppt
Topic 60 CERVICAL RIB.pptTopic 60 CERVICAL RIB.ppt
Topic 60 CERVICAL RIB.ppt
 
Maxillofacial prosthesis
Maxillofacial prosthesisMaxillofacial prosthesis
Maxillofacial prosthesis
 
DR SAGAR.pptx
DR SAGAR.pptxDR SAGAR.pptx
DR SAGAR.pptx
 
Twin block appliance.
Twin block appliance.Twin block appliance.
Twin block appliance.
 
Sp06 starfish parts 1
Sp06 starfish parts 1Sp06 starfish parts 1
Sp06 starfish parts 1
 
Principles_of_fractures-1.ppt
Principles_of_fractures-1.pptPrinciples_of_fractures-1.ppt
Principles_of_fractures-1.ppt
 

Recently uploaded

一比一原版Otago毕业证成绩单如何办理
一比一原版Otago毕业证成绩单如何办理一比一原版Otago毕业证成绩单如何办理
一比一原版Otago毕业证成绩单如何办理cnzepoz
 
Abortion pills in Riyadh |•••@•••| +966572737505 |•••@•••| Buy Cytotec
Abortion pills in Riyadh |•••@•••| +966572737505 |•••@•••| Buy CytotecAbortion pills in Riyadh |•••@•••| +966572737505 |•••@•••| Buy Cytotec
Abortion pills in Riyadh |•••@•••| +966572737505 |•••@•••| Buy CytotecAbortion pills in Riyadh +966572737505 get cytotec
 
Aluminum Die Casting Manufacturers in China - BIAN Diecast
Aluminum Die Casting Manufacturers in China - BIAN DiecastAluminum Die Casting Manufacturers in China - BIAN Diecast
Aluminum Die Casting Manufacturers in China - BIAN DiecastAMshares
 
一比一原版迪肯大学毕业证成绩单如何办理
一比一原版迪肯大学毕业证成绩单如何办理一比一原版迪肯大学毕业证成绩单如何办理
一比一原版迪肯大学毕业证成绩单如何办理cnzepoz
 
NO1 Qari Rohani Amil In Islamabad Amil Baba in Rawalpindi Kala Jadu Amil In R...
NO1 Qari Rohani Amil In Islamabad Amil Baba in Rawalpindi Kala Jadu Amil In R...NO1 Qari Rohani Amil In Islamabad Amil Baba in Rawalpindi Kala Jadu Amil In R...
NO1 Qari Rohani Amil In Islamabad Amil Baba in Rawalpindi Kala Jadu Amil In R...Amil baba
 
一比一原版UBC毕业证成绩单如何办理
一比一原版UBC毕业证成绩单如何办理一比一原版UBC毕业证成绩单如何办理
一比一原版UBC毕业证成绩单如何办理cnzepoz
 
Balancing of rotating bodies questions.pptx
Balancing of rotating bodies questions.pptxBalancing of rotating bodies questions.pptx
Balancing of rotating bodies questions.pptxjoshuaclack73
 
一比一原版UVic毕业证成绩单如何办理
一比一原版UVic毕业证成绩单如何办理一比一原版UVic毕业证成绩单如何办理
一比一原版UVic毕业证成绩单如何办理cnzepoz
 
China Die Casting Manufacturer & Supplier - Bian Diecast
China Die Casting Manufacturer & Supplier - Bian DiecastChina Die Casting Manufacturer & Supplier - Bian Diecast
China Die Casting Manufacturer & Supplier - Bian DiecastAMshares
 
一比一原版麦考瑞大学毕业证成绩单如何办理
一比一原版麦考瑞大学毕业证成绩单如何办理一比一原版麦考瑞大学毕业证成绩单如何办理
一比一原版麦考瑞大学毕业证成绩单如何办理cnzepoz
 
一比一原版SUT毕业证成绩单如何办理
一比一原版SUT毕业证成绩单如何办理一比一原版SUT毕业证成绩单如何办理
一比一原版SUT毕业证成绩单如何办理cnzepoz
 
一比一原版UofM毕业证成绩单如何办理
一比一原版UofM毕业证成绩单如何办理一比一原版UofM毕业证成绩单如何办理
一比一原版UofM毕业证成绩单如何办理cnzepoz
 
一比一原版UW毕业证成绩单如何办理
一比一原版UW毕业证成绩单如何办理一比一原版UW毕业证成绩单如何办理
一比一原版UW毕业证成绩单如何办理cnzepoz
 
一比一原版UMich毕业证成绩单如何办理
一比一原版UMich毕业证成绩单如何办理一比一原版UMich毕业证成绩单如何办理
一比一原版UMich毕业证成绩单如何办理cnzepoz
 
一比一原版ArtEZ毕业证成绩单如何办理
一比一原版ArtEZ毕业证成绩单如何办理一比一原版ArtEZ毕业证成绩单如何办理
一比一原版ArtEZ毕业证成绩单如何办理cnzepoz
 
NO1 Qari kala jadu karne wale ka contact number kala jadu karne wale baba kal...
NO1 Qari kala jadu karne wale ka contact number kala jadu karne wale baba kal...NO1 Qari kala jadu karne wale ka contact number kala jadu karne wale baba kal...
NO1 Qari kala jadu karne wale ka contact number kala jadu karne wale baba kal...Amil baba
 
一比一原版Southern Cross毕业证成绩单如何办理
一比一原版Southern Cross毕业证成绩单如何办理一比一原版Southern Cross毕业证成绩单如何办理
一比一原版Southern Cross毕业证成绩单如何办理cnzepoz
 
一比一原版UC Berkeley毕业证成绩单如何办理
一比一原版UC Berkeley毕业证成绩单如何办理一比一原版UC Berkeley毕业证成绩单如何办理
一比一原版UC Berkeley毕业证成绩单如何办理cnzepoz
 
一比一原版UCB毕业证成绩单如何办理
一比一原版UCB毕业证成绩单如何办理一比一原版UCB毕业证成绩单如何办理
一比一原版UCB毕业证成绩单如何办理cnzepoz
 
一比一原版AIS毕业证成绩单如何办理
一比一原版AIS毕业证成绩单如何办理一比一原版AIS毕业证成绩单如何办理
一比一原版AIS毕业证成绩单如何办理cnzepoz
 

Recently uploaded (20)

一比一原版Otago毕业证成绩单如何办理
一比一原版Otago毕业证成绩单如何办理一比一原版Otago毕业证成绩单如何办理
一比一原版Otago毕业证成绩单如何办理
 
Abortion pills in Riyadh |•••@•••| +966572737505 |•••@•••| Buy Cytotec
Abortion pills in Riyadh |•••@•••| +966572737505 |•••@•••| Buy CytotecAbortion pills in Riyadh |•••@•••| +966572737505 |•••@•••| Buy Cytotec
Abortion pills in Riyadh |•••@•••| +966572737505 |•••@•••| Buy Cytotec
 
Aluminum Die Casting Manufacturers in China - BIAN Diecast
Aluminum Die Casting Manufacturers in China - BIAN DiecastAluminum Die Casting Manufacturers in China - BIAN Diecast
Aluminum Die Casting Manufacturers in China - BIAN Diecast
 
一比一原版迪肯大学毕业证成绩单如何办理
一比一原版迪肯大学毕业证成绩单如何办理一比一原版迪肯大学毕业证成绩单如何办理
一比一原版迪肯大学毕业证成绩单如何办理
 
NO1 Qari Rohani Amil In Islamabad Amil Baba in Rawalpindi Kala Jadu Amil In R...
NO1 Qari Rohani Amil In Islamabad Amil Baba in Rawalpindi Kala Jadu Amil In R...NO1 Qari Rohani Amil In Islamabad Amil Baba in Rawalpindi Kala Jadu Amil In R...
NO1 Qari Rohani Amil In Islamabad Amil Baba in Rawalpindi Kala Jadu Amil In R...
 
一比一原版UBC毕业证成绩单如何办理
一比一原版UBC毕业证成绩单如何办理一比一原版UBC毕业证成绩单如何办理
一比一原版UBC毕业证成绩单如何办理
 
Balancing of rotating bodies questions.pptx
Balancing of rotating bodies questions.pptxBalancing of rotating bodies questions.pptx
Balancing of rotating bodies questions.pptx
 
一比一原版UVic毕业证成绩单如何办理
一比一原版UVic毕业证成绩单如何办理一比一原版UVic毕业证成绩单如何办理
一比一原版UVic毕业证成绩单如何办理
 
China Die Casting Manufacturer & Supplier - Bian Diecast
China Die Casting Manufacturer & Supplier - Bian DiecastChina Die Casting Manufacturer & Supplier - Bian Diecast
China Die Casting Manufacturer & Supplier - Bian Diecast
 
一比一原版麦考瑞大学毕业证成绩单如何办理
一比一原版麦考瑞大学毕业证成绩单如何办理一比一原版麦考瑞大学毕业证成绩单如何办理
一比一原版麦考瑞大学毕业证成绩单如何办理
 
一比一原版SUT毕业证成绩单如何办理
一比一原版SUT毕业证成绩单如何办理一比一原版SUT毕业证成绩单如何办理
一比一原版SUT毕业证成绩单如何办理
 
一比一原版UofM毕业证成绩单如何办理
一比一原版UofM毕业证成绩单如何办理一比一原版UofM毕业证成绩单如何办理
一比一原版UofM毕业证成绩单如何办理
 
一比一原版UW毕业证成绩单如何办理
一比一原版UW毕业证成绩单如何办理一比一原版UW毕业证成绩单如何办理
一比一原版UW毕业证成绩单如何办理
 
一比一原版UMich毕业证成绩单如何办理
一比一原版UMich毕业证成绩单如何办理一比一原版UMich毕业证成绩单如何办理
一比一原版UMich毕业证成绩单如何办理
 
一比一原版ArtEZ毕业证成绩单如何办理
一比一原版ArtEZ毕业证成绩单如何办理一比一原版ArtEZ毕业证成绩单如何办理
一比一原版ArtEZ毕业证成绩单如何办理
 
NO1 Qari kala jadu karne wale ka contact number kala jadu karne wale baba kal...
NO1 Qari kala jadu karne wale ka contact number kala jadu karne wale baba kal...NO1 Qari kala jadu karne wale ka contact number kala jadu karne wale baba kal...
NO1 Qari kala jadu karne wale ka contact number kala jadu karne wale baba kal...
 
一比一原版Southern Cross毕业证成绩单如何办理
一比一原版Southern Cross毕业证成绩单如何办理一比一原版Southern Cross毕业证成绩单如何办理
一比一原版Southern Cross毕业证成绩单如何办理
 
一比一原版UC Berkeley毕业证成绩单如何办理
一比一原版UC Berkeley毕业证成绩单如何办理一比一原版UC Berkeley毕业证成绩单如何办理
一比一原版UC Berkeley毕业证成绩单如何办理
 
一比一原版UCB毕业证成绩单如何办理
一比一原版UCB毕业证成绩单如何办理一比一原版UCB毕业证成绩单如何办理
一比一原版UCB毕业证成绩单如何办理
 
一比一原版AIS毕业证成绩单如何办理
一比一原版AIS毕业证成绩单如何办理一比一原版AIS毕业证成绩单如何办理
一比一原版AIS毕业证成绩单如何办理
 

Orthopedics Class Day 7.ppt

  • 1. Dreamz Learning Innovations_____________________________________________ Page 1 41. Migrating (wandering) acetabulum is seen in A. Fracture dislocation of B. Acetabulum in advanced stage of Tuberculous hip C. proximal femoral deficiency D. perthes' disease
  • 2. Dreamz Learning Innovations_____________________________________________ Page 2 41. Migrating (wandering) acetabulum is seen in A. Fracture dislocation of B. Acetabulum in advanced stage of Tuberculous hip C. proximal femoral deficiency D. perthes' disease B
  • 3. Dreamz Learning Innovations_____________________________________________ Page 3 Radiological Features To begin with there is reduction in the joint space with juxta-artcular osteopenia. In advanced stage of arthritis the radiological appearance of hip has been classified by Prof TK Shanmuga sundaram into: 1. Normal type 2. Wandering acetabulum: There occurs destruction of the acetabulum in its superior due to the disease. The temporal head shifts proximally on the ilium. This is called wandering acetabulum.
  • 4. Dreamz Learning Innovations_____________________________________________ Page 4 3. Dislocated acetabulum. 4. Pertheoid type: The radiographic picture looks somewhat similar to that of the Perthes disease. 5. Atrophic type. 6. Protrusio acetabuli.
  • 5. Dreamz Learning Innovations_____________________________________________ Page 5 42. Radial nerve is known to be involved in which of the following injuries A. Fracture neck of Humerus B. Fracture shaft of Humerus , lower third C. Fracture Humerus distal end D. Dislocation of elbow
  • 6. Dreamz Learning Innovations_____________________________________________ Page 6 42. Radial nerve is known to be involved in which of the following injuries A. Fracture neck of Humerus B. Fracture shaft of Humerus , lower third C. Fracture Humerus distal end D. Dislocation of elbow B
  • 7. Dreamz Learning Innovations_____________________________________________ Page 7 43. Which of the following fracture is best treated by surgery? A.Fracture Intertrochanteric B.Fracture neck of femur C.Fracture tibial condyles D. Fracture Scaphoid
  • 8. Dreamz Learning Innovations_____________________________________________ Page 8 43. Which of the following fracture is best treated by surgery? A.Fracture Intertrochanteric B.Fracture neck of femur C.Fracture tibial condyles D. Fracture Scaphoid B
  • 9. Dreamz Learning Innovations_____________________________________________ Page 9 Femoral neck fractures usually require surgery for treatment
  • 10. Dreamz Learning Innovations_____________________________________________ Page 10 44. Osteoporosis is characterized by A. Decreased mineralization of bone B. Decrease in marrow content of bone C. Decrease in bone mass D. Softening and deformity of bone
  • 11. Dreamz Learning Innovations_____________________________________________ Page 11 44. Osteoporosis is characterized by A. Decreased mineralization of bone B. Decrease in marrow content of bone C. Decrease in bone mass D. Softening and deformity of bone C
  • 12. Dreamz Learning Innovations_____________________________________________ Page 12 45. Sequestmm is a A. Living bone formed in Osteomyelitis B. Dead infected bone formed in Osteomyelitis C. Bony thickening felt in Osteomyelitis D. New bone formed in soft tissue planes
  • 13. Dreamz Learning Innovations_____________________________________________ Page 13 45. Sequestmm is a A. Living bone formed in Osteomyelitis B. Dead infected bone formed in Osteomyelitis C. Bony thickening felt in Osteomyelitis D. New bone formed in soft tissue planes B
  • 14. Dreamz Learning Innovations_____________________________________________ Page 14 46. In case of Poliomyelitis, 'residual stage' of paralysis is called after A. 3 months of onset of acute Poliomyelitis B. 6 months of onset of acute Poliomyelitis C. 12 months of onset of acute Poliomyelitis D. 24 months of onset of acute Poliomyelitis
  • 15. Dreamz Learning Innovations_____________________________________________ Page 15 46. In case of Poliomyelitis, 'residual stage' of paralysis is called after A. 3 months of onset of acute Poliomyelitis B. 6 months of onset of acute Poliomyelitis C. 12 months of onset of acute Poliomyelitis D. 24 months of onset of acute Poliomyelitis D
  • 16. Dreamz Learning Innovations_____________________________________________ Page 16 Residual Paralysis: As the acute phase of illness (0-4 weeks) subsides, the recovery begins in paralyzed muscles. The extent of recovery is variable ranging from mild to severe residual paresis at 60 days, depending upon the extent of damage caused to the neurons by the virus. Maximum neurological recovery of the paralyzed muscle takes place in the first six months of the illness but slow recovery continues up to two years. After two years, no more recovery is expected and the child is said to have “Post Polio residual paralysis”, which remains as such through out life. However, the child can learn to use muscles which were not paralyzed to compensate for lost muscle power.
  • 17. Dreamz Learning Innovations_____________________________________________ Page 17 47. Syme's Amputation is done through A. Ankle joint B. Knee joint C. Hip joint D. Lower third of tibia
  • 18. Dreamz Learning Innovations_____________________________________________ Page 18 47. Syme's Amputation is done through A. Ankle joint B. Knee joint C. Hip joint D. Lower third of tibia A
  • 19. Dreamz Learning Innovations_____________________________________________ Page 19 Syme's Amputation includes ankle disarticulation, removal of malleoli, & anchoring heel pad to the wt bearing surface) allows execellent gait with a cosmetic prosthesis; - symes amputations will not heal w/o palpable posterior artery pulse; - surgery may be performed in 2 stages; - most common indication for this is infection; - ankle is disarticulated in the first stage and the amputation is revised approximately 6 weeks later during the second stage;
  • 20. Dreamz Learning Innovations_____________________________________________ Page 20 - components of the 2nd stage include resection of the malleoli flush w/ joint surface, fixation of the fat pad to residual bone, and revision of redundant skin; - this amputation allows for intermittent wt bearing; however, skin break down may occur if a prosthesis is used on a regular basis; - majority of patients w/ Syme's amputation will require prothesis designed to relieve end weight bearing;
  • 23. Dreamz Learning Innovations_____________________________________________ Page 23 48. Indication of open amputation is A.Dry gangrene B.Buerger’s gangrene C.Gas gangrene D.Rayrnaud's disease
  • 24. Dreamz Learning Innovations_____________________________________________ Page 24 48. Indication of open amputation is A.Dry gangrene B.Buerger’s gangrene C.Gas gangrene D.Rayrnaud's disease C
  • 25. Dreamz Learning Innovations_____________________________________________ Page 25 Most surgeons agree that early and adequate surgery is the most effective and primary means of treating clostridial myositis. If the diagnosis is made early, while the gangrene is relatively localized and incipient, radical decompression of the involved fascial compartments by free longitudinal incisions and excision of the infected muscle usually arrest the process and eliminate the need for amputation. Local excision preferred to amputaion, whenever posibie, to conserve a functional extremity. If diagnosis is reached when the process is extensive and has caused irreversible gangrenous changes implying permanent loss of function of the limp, open amputation of the guillotine type or some modification becomes necessary.
  • 26. Dreamz Learning Innovations_____________________________________________ Page 26 49. Fracture of lower end of Radius is also called A. Bennett’s fracture B. Colles’ fracture C. Jones’ fracture D. Boxer’s fracture
  • 27. Dreamz Learning Innovations_____________________________________________ Page 27 49. Fracture of lower end of Radius is also called A. Bennett’s fracture B. Colles’ fracture C. Jones’ fracture D. Boxer’s fracture B
  • 28. Dreamz Learning Innovations_____________________________________________ Page 28 A Jones fracture is a fracture of the diaphysis of the fifth metatarsal of the foot. The fifth metatarsal is at the base of the small toe.
  • 29. Dreamz Learning Innovations_____________________________________________ Page 29 Bennett's fracture is a fracture of the base of the first metacarpal bone which extends into the carpometacarpal (CMC) joint. This intra-articular fracture
  • 30. Dreamz Learning Innovations_____________________________________________ Page 30 A "Boxer's fracture" is the second and/or third metacarpal transverse neck fracture that is more likely to occur from a straight punch. The "Boxer's" designation is suggestive of the generally well tolerated way of striking a hard object with the closed fist, with the second and third metacarpal bones, of the index and middle fingers respectively, absorbing most of the force.
  • 31. Dreamz Learning Innovations_____________________________________________ Page 31 50. The primary site of infection in Pott’s spine is A. Vertebral body B. inter-vertebral disc C. Transverse process D. Spinous process
  • 32. Dreamz Learning Innovations_____________________________________________ Page 32 50. The primary site of infection in Pott’s spine is A. Vertebral body B. inter-vertebral disc C. Transverse process D. Spinous process B
  • 33. Dreamz Learning Innovations_____________________________________________ Page 33 Pott disease (also known as tuberculous spondylitis) refers to vertebral body and intervertebral disc involvement with tuberculosis
  • 34. Dreamz Learning Innovations_____________________________________________ Page 34 51. Most sensitive test for Anterior cruciate ligament tear A. Apley's grinding test B. Drawer test C. Lachman test D. Me murray's test
  • 35. Dreamz Learning Innovations_____________________________________________ Page 35 51. Most sensitive test for Anterior cruciate ligament tear A. Apley's grinding test B. Drawer test C. Lachman test D. Me murray's test C
  • 36. Dreamz Learning Innovations_____________________________________________ Page 36 LACHMAN-TEST - is the most sensitive test for ACL tear It is done with the knee flexed at 20 degree, stablizing the distal femur with one hand and pulling forward on the proximal tibia with the other hand with an intact ligament minimal translation of the tibia occurs and a firm end point is felt with a torn ACL more translation is noted and the end point is soft or mushy • The blood supply of ACL is primarily derived from Middle genicular artery***
  • 37. Dreamz Learning Innovations_____________________________________________ Page 37 ANTERIOR -DRAWER TEST - is done with the knee at 90° degree of flexion and is not as sensitive as Lachman test but serves as an adjunct in the evaluation of ACL instability LOSEE TEST - THE PIVOT- SHIFT- PHENOMENON Demonstrate the instability associated with an ACL tear,
  • 38. Dreamz Learning Innovations_____________________________________________ Page 38 'As described by losee, a valgus and internal rotation force is applied to the tibia. Starting at 45 degree of flexion. The lateral tibial plateau is reduced. Extending the knee causes the lateral plateau to subluxate anteriorly with a thud at approximately 20 degrees of flexion. It reduces quietly at full extension. Posterior drawer test - evaluates the integrity of the PCL MC- Murray test - with the Me Murray test forced flexion and rotation of the knee elicits a clunk along the joint line if there is meniscus injury** Investigation of choice in Knee joint injury is MRI
  • 39. Dreamz Learning Innovations_____________________________________________ Page 39 Anatomic correlation of clinical ligament instability examination of the knee joint Direction of force Position Ligament instability Varus or valgus Full extension Posterior cruciate/ posterior capsule Varus Flexion at 30 degree Lateral collateral ligament /capsule Valgus Flexion at 30 degree Medial collateral ligament Anterior Flexion at 30° neutral position (AP) Anterior cruciate ligament Anterior Flexion at 90° neutral internal or external rotation Anterior cruciate ligament Posterior 90° (Sag test) Posterior cruciate ligament
  • 40. Dreamz Learning Innovations_____________________________________________ Page 40 Important Points • Medial meniscus has less mobility than the lateral meniscus and is more susceptible tearing when trapped between the femoral condyle and tibial plateau • Anterior cruciate ligament prevents anterior translation and rotation of the tibia on the femur, • Posterior cruciate ligament prevents posterior subluxation of the tibia on the femur • Posterior glenohumeral instability can be tested by Jerk Test • Coronary ligament of the knee axe —ligaments connecting the menisci to tibia***
  • 41. Dreamz Learning Innovations_____________________________________________ Page 41 52. Sudden forceful rapture of the quadriceps femoris muscle will lead to rupture of A. Vastus intermedius B. Vastus lateralis C. Rectus femoris D. Vastus mediali's
  • 42. Dreamz Learning Innovations_____________________________________________ Page 42 52. Sudden forceful rapture of the quadriceps femoris muscle will lead to rupture of A. Vastus intermedius B. Vastus lateralis C. Rectus femoris D. Vastus mediali's C
  • 43. Dreamz Learning Innovations_____________________________________________ Page 43 Tear of Quadriceps Tendon - occurs most often in patients over the age of 40, apparent tears that represent avulsions from the patella occur in patients with renal osleodystrophy or hyperparathyroidism. The tear commonly results from sudden deceleration, such as stumbling or stipping on a wet surface. A small flake of bone may be avulsed from the superior pole of the patella or the tear may occur entirely through tendinous and muscular tissues Quadriceps rupture at the upper margin of the patella - This is commonly known as an avulsion of the rectus femoris. It occurs in elderly patients. It is diagnosed by the palpation of a gap in the extensor apparatus just above the patella two fingers can be placed in the gap usually the lateral expansions are intact (Watson & Jones).
  • 44. Dreamz Learning Innovations_____________________________________________ Page 44 Muscles Actions • Rectus femoris • Extension of leg at knee joint flexes thigh at hip joint*** • Vastus medialis • Extension of leg at knee joint: Stabilies patella • Vastus intermedius • Extension of leg at knee joint: articularis genus retracts synovial membrane • Vastus lateralis • Extension of leg at knee joint • Biceps femoris • Chief flexors of knee. They are weak extensors of the hip particularly in walking • The rectus femoris muscle can rupture in sudden violent extension movement of the knee joints** • Vastus medialis is the first part of the quadriceps muscle to atrophy in knee joint and the last to recover***
  • 45. Dreamz Learning Innovations_____________________________________________ Page 45 53. Von- rosen splint are applicable in A. Developmental dysplasia of hip joint B. Congenital talipes equinovarus C. Congenital vertical talus D. Tibia mileruae
  • 46. Dreamz Learning Innovations_____________________________________________ Page 46 53. Von- rosen splint are applicable in A. Developmental dysplasia of hip joint B. Congenital talipes equinovarus C. Congenital vertical talus D. Tibia mileruae A
  • 47. Dreamz Learning Innovations_____________________________________________ Page 47 VON-ROSEN'S SPLINT - is an H-shaped melleable splint • Object of splintage is to hold the hips some what flexed and abducted • Used in congenital dislocation / dysplasia of hip joint Also - Remember • Frog leg or Lorenz cast or Batchelor cast used in CDH • Broomstick plaster used in perthe's disease • Patella Tendon Bearing (PTB) cast used in- Both bones leg fracture
  • 48. Dreamz Learning Innovations_____________________________________________ Page 48 • Gutter splint- phalyngeal and metaphalyngeal fractures • Denis Browne splint used in CTEV, Stirrup splint Ankle fractures • Knuckle-bender splint- ulner nerve palsy, Cock up splint - Radial nerve palsy • Milwaukee brace (Cervical thoracolombosacral orthosis) used in Scoliosis
  • 49. Dreamz Learning Innovations_____________________________________________ Page 49 • Risser's localizer cast, Turn buckle cast- Scoliosis • Minerva cast - cerical spine disease • U slab or Hanging cast - fracture of humerus • Tube cast (Cylindrical cast) - used for fracture around knee (# patella) • Halo cast (vest) - for spinal injury • Velpeau bandage and sling and Swathe splint are used in - Shoulder dislocation** • Thromboembolic disease (Pulmonary embolism) is the most common cause of death after Total hip replacement*** (Watson jones operation)
  • 50. Dreamz Learning Innovations_____________________________________________ Page 50 • Pectorafis major and minor muscles are the most common congenitically absent muscles in humans** • Pollicization refer to reconstruction of the thumb with a substitute finger (i.e. index finger) • Bohler's Tuber Joint angle and crucial angle of Gissane are measured for intraarticular # ofcalcanium Development Dysplasia of the hep (DDH) • More common in female, short stature** • Oligohydramnios is associated with a higher risk of DDH
  • 51. Dreamz Learning Innovations_____________________________________________ Page 51 • The hourglass appearance of the capsule may prevent a successful closed reduction • In a child with bilateral dislocation - Waddling gait- unilateral dislocation Trendelenburg's gait* • X-ray shows break in shenton's line* • Striking feature is widening of perineum & marked lumber lorodosis* • Wandering acetabulum is sen in T.B of hip joints
  • 52. Dreamz Learning Innovations_____________________________________________ Page 52 Angles of Importance (i) Cobb's angle - Scoliosis (ii) Kite's angle - CTEV (iii) Meary's angle - Pes cavus (iv) Hilagenreiner's epiphyseal angle - congenital coxa-vera (v) Baumann's angle - Supracondylar # Rocker bottom foot or vertical talus is associated with Arthrogrypasis multiplex congenital spina bifida & trisomy 13-18
  • 53. Dreamz Learning Innovations_____________________________________________ Page 53 54. Congenital dislocation of hip is common in A. Chinese B. Asian C. Negro D. North American Indians
  • 54. Dreamz Learning Innovations_____________________________________________ Page 54 54. Congenital dislocation of hip is common in A. Chinese B. Asian C. Negro D. North American Indians D
  • 55. Dreamz Learning Innovations_____________________________________________ Page 55 CONGENITAL - DISLOCATION OF THE HIP (CDH) - In western races. It is one of the commonest congenital disorder but it is uncommon in India and some other Asian countries probably because of the culture of mother carrying the child on the side of their waist with the hips of the child abducted. This position helps in the reduction of an unstable hip which otherwise would have dislocated (200-M) • Dislocation is very common in Lapps and North American Indians who swaddle their babies and carry them with legs together, hips and knees fully extended and is rare in southern Chinese and African Negroes who carry their babies astride their backs with legs widely abducted (Apley)
  • 56. Dreamz Learning Innovations_____________________________________________ Page 56 • Investigation of choice in newborn is real time ultrasound Aetiology and pathogencsis • Genetic factor - dominant trait, and acetabular dysplasia • Hormonal factors - High levels of maternal oestrogen progesterone and relaxin • Breach position with extended legs • Postnatal factors - contribute to persistence of neonatal instability and acetabular maldevelopment
  • 57. Dreamz Learning Innovations_____________________________________________ Page 57 Diagnosis of CDH • Limitation of the abduction of the hip • Ortoloni'test is positive (Jerk of entry') • Asymmetrical thigh fold • Barlow's test • High buttock fold on the affected side • Telescopy positive • Galeazzi 's sign • Trendelenburg, waddling gait* • Limb is short and slightly externally rotated • Trendelenburg's test positive
  • 58. Dreamz Learning Innovations_____________________________________________ Page 58 Important X-ray findings in CDH - Delayed appearance of ossific centre for the head of the femur . - Retarded development of ossific centre of the head of the femur - Slopping acetabulum - Lateral and upward displacement of the ossific centre of the femoral head - A break in shenton 's line***
  • 59. Dreamz Learning Innovations_____________________________________________ Page 59 55. Most common organism in drug abusers chronic osteomyelitis is A. Staph.aureus B. Pseudomonas C. Granulmatous D. Klebsiella
  • 60. Dreamz Learning Innovations_____________________________________________ Page 60 55. Most common organism in drug abusers chronic osteomyelitis is A. Staph.aureus B. Pseudomonas C. Granulmatous D. Klebsiella B
  • 61. Dreamz Learning Innovations_____________________________________________ Page 61 Most common organisms in Osteomyelitis • Acute osteomyelitis- children under 4 years of age. H. influenme • Acute osteomyelitis in older children, adults - Staph.aureus • Sickle cell disease - salmonella • Heroin addicts and immunocopromised—Pseudomonas, aeniginosa, proteus mirabilis, Bacteroids • Sub acute haematogenous osteomyelitis - Staph. aureus • Post traumatic osteomyelitis, Infected metal implants Staph. aureus
  • 62. Dreamz Learning Innovations_____________________________________________ Page 62 • Post operative osteomyelitis - Mixed infection (Staph. aureus Proteus, Pseudomonas) • Osteomyelitis of the spine - Staph, aureus, Pseudomonas aeruginosa • Squamous cell carcinoma arising from a chronic osteomyelitis - occurs in areas of burn scars, chronic pressure ulcers, and ostomies as well as as sites of chronic draining osteomyelitis Stable chronic osteomyelitis for an average of approximately 20 years
  • 63. Dreamz Learning Innovations_____________________________________________ Page 63 Earliest site of bone involvement in hematogenous osteomyelitis is - Metaphysis** Markers of Bone formation Markers of Bone resorption • Serum bone specific alkaline phosphatase** • Serum osteocalcin • Serum propeptide or type I procollagen • Urine and serum N- telopeptide • Urine and serum C- telopeptide* • Urine total free deoxypyridinoline • Urine Hydroxyproline* • Serum tartarate resistant- Acid phosphatase* • Serum bone sialoprtein • Urine hydroxylysine glycosides
  • 64. Dreamz Learning Innovations_____________________________________________ Page 64 • Infective endocarditis due to pseudomonas is most commonly seen with IV drug abuse of pentazocin** Classification systems for Osteomyelitis 1. Traditional system - Acute (<2 weeks) subacute (weeks to months) chronic (>3months) 2. Waldvogel system - Hematogenous - Arising from contiguous infection No vascular disease - Vascular disease present chronic 3. Cierny- Mader system Anatomic extent of infection 1. Medullary only (acute hematogenous) 2. Superficial cortex (continguous spread or soft tissue trauma)
  • 65. Dreamz Learning Innovations_____________________________________________ Page 65 3. Localized (cortical and medullary, mechanically stable) 4. Diffuse (cortical and medullary mechanically unstable) 5. Subtype by host's physiologic status A Healty Bs Compromised because of systemic factors B1 Compromised because of local factors B1s Compromised because of both local and systemic factors C Treatment worse than the disease
  • 66. Dreamz Learning Innovations_____________________________________________ Page 66 56. A-man is stucked with lathi,below the knee in lateral aspect of the leg. He drags the toes and cannot feel the sensation in dorsum of feet which of the following is intact A. Inversion B. Eversion C. Dorsiflexion D. Loss of sensation in 1st & 2nd toes
  • 67. Dreamz Learning Innovations_____________________________________________ Page 67 56. A-man is stucked with lathi,below the knee in lateral aspect of the leg. He drags the toes and cannot feel the sensation in dorsum of feet which of the following is intact A. Inversion B. Eversion C. Dorsiflexion D. Loss of sensation in 1st & 2nd toes A
  • 68. Dreamz Learning Innovations_____________________________________________ Page 68 Clinical picture suggest diagnosis is a lateral popliteal nerve injury • Causes of Lateral popliteal nerve palsy (Waston & Jones') • Rupture of the external lateral ligament • Avulsion of the styloid process of the fibula • Dislocation of the knee joint • Compressed by. Splint, POP or strapping • Ganglion arising from superior tibio-fibular joint • Cyst of the lateral meniscus
  • 69. Dreamz Learning Innovations_____________________________________________ Page 69 Clinical features • Paralysis of the anterior tibial and peroneal muscle • Loss of sensibility in the outer aspect of the leg and dorsum of the foot PERONEAL NERVE (Apley) • Injuries may affect either the common peroneal {lateralpopliteal) nerve or one of its branches the deep or superficial peroneal nerve
  • 70. Dreamz Learning Innovations_____________________________________________ Page 70 [A]. Common peroneal nerve (Lateral popliteal nerve) • The patient has drop foot and can neither dorsifiex nor evert the foot** • He walks with a high stepping gait to avoid catching the toes • Sensation is lost over the front and outer half of the leg and the dorsum of the foot** • Planter flexion & Inversion are normal**
  • 71. Dreamz Learning Innovations_____________________________________________ Page 71 [B]. Deep peroneal nerve • Pain and weakness of dorsiflexion • Sensory loss in a small area of skin between the first and second toes** • Paraesthesia and numbness on the dorsum around the first web space [C]. Superficial peroneal nerve • Pain in the lateral part of the leg and numbness or paraesthesia of the foot • There may be weakness of eversion and sensory loss on the dorsum of the foot • Most commonly injured in misplaced gluteal injection - Sciatic nerve injury • Most common cause of insertional tendonitis of tendoachilis is - over use**
  • 72. Dreamz Learning Innovations_____________________________________________ Page 72 57. True statement regarding posterior dislocation of shoulder joint A. External rotation B. Internal rotation C. Axillary nerve palsy D. Anterior hollowness
  • 73. Dreamz Learning Innovations_____________________________________________ Page 73 57. True statement regarding posterior dislocation of shoulder joint A. External rotation B. Internal rotation C. Axillary nerve palsy D. Anterior hollowness B
  • 74. Dreamz Learning Innovations_____________________________________________ Page 74 • Anterior dislocation is much more common than posterior Posterior dislocation of Schoulder is rare Mechanism of injury — Indirect force producing marked internal rotation* and adduction* must needs be very sever to cause a dislocation. This happens most commonly during a fit or convulsions or with electric shock.
  • 75. Dreamz Learning Innovations_____________________________________________ Page 75 ANTERIOR SHOULDER DlSLOCATION POSTERIOR SHOULDER DISLOCATION • Due to shallowness of the glenoid socket, the extra ordinary range of the movement, ligamentous laxity or glenoid dysplasia • The lateral out line of the shoulder may be flattened, bulge may be felt just below the clavicle • The front of the shoulder looks flat with a prominent coracoid • Posterior displacement is usually apparent • The characteristic physical sign is loss of lateral rotation of the shoulder which is locked in medial rotation (Watson & Jones) • Posterior dislocations are often difficult to diagnose because the patient may have a normal contour to the shoulder or the deltoid of a well developed athlete may mask signs of a displaced humeral head (207-current diagnosis & treatment 4th) • A "reverse Hill- Sachs lesion" may appear on the anterior articular surface of the humerus Patho physiological changes • Bankart's lesion — Dislocation causes stripping of the glenoidal labrum along with the periosteum from the anterior surface of the glenoid and scapular neck*** • Hill- Sach 's lesion — is a depression on the humeral head in its postero-lateral quadrant. • Eburnated rim of the glenoid • Associated # of the GT of the humerus or rim of the glenoid
  • 76. Dreamz Learning Innovations_____________________________________________ Page 76 • Dugas' test, Hamilton ruler test, callway's test are positive in Anterior dislocation of the shoulder joint • Axillary nerve and Axillary artery injury is common in Anterior dislocation of the shoulder joint Empty glenoid sign - in AP- film the humeral head because it is medially rotated, looked abnormal in shape (like an electric light bulb) and it stands away somewhat from glenoid fossa is seen in posterior dislocation of the shoulder joint • Patella almost always dislocates laterally • Recurrent dislocations are extremely rare in the Ankle joints*
  • 77. Dreamz Learning Innovations_____________________________________________ Page 77 Recurrent dislocations of shoulder joints Three important lesions 1. Hill sach's lesions** 2. Bankart's lesion** 3. Erosion of anterior rim of glenoid cavity Oilier pathologic lesions 1. Capsular laxity* 2. Sub capsular's deficiency 3. Glenoid fossa deficiency
  • 78. Dreamz Learning Innovations_____________________________________________ Page 78 Anterior cruciate ligament Posterior cruciate ligament • Prevent anterior translation of the tibia on the femur**.limits hyper extension • Attached on tibia just behind anterior horn of medial meniscus Runs upwards bat Km aids and laterally • Attached on femur over medial surface of lateral femoral condyle • It is intracapsular and intrasynoviai* • It is tout during extension of knee • Prevent posterior translation on tibia on the femur** • Attached on tibia behind posterior horn of medialmeniscus Runs upwards forwards and medially • Attached on femur over lateral surface of medial femoral condyle • It is intracapsular but extrasynovial* • It is tout during flexion of knee*
  • 79. Dreamz Learning Innovations_____________________________________________ Page 79 58. Following are reduce the infection rate in implant surgery EXCEPT A. Laminar flow B. Low air flow C. Body exaust custom D. Frequent hand washing
  • 80. Dreamz Learning Innovations_____________________________________________ Page 80 58. Following are reduce the infection rate in implant surgery EXCEPT A. Laminar flow B. Low air flow C. Body exaust custom D. Frequent hand washing B
  • 81. Dreamz Learning Innovations_____________________________________________ Page 81 Cohart placement in an area with an independent air supply, exhaust system & bathroom facilities • Surgeons may use double gloves, space suits adherent plastic drapes, pulstile lavage, and laminar flow operating rooms or rooms with ultraviolet lights as additional techniques that may lower the risk of intraoperative contamination The main preventive measures of Infection control are 1. Isolation - Infectious patients must be isolated 2. Hospital staff- Those who are suffering from skin diseases, sore throat, common cold, ear infection diarrhea or dysentery and other infectious ailments should be kept away from work until completely cured
  • 82. Dreamz Learning Innovations_____________________________________________ Page 82 3. Hand washing - The most common route of infection is via the hands. Hand washing with soap and water may not be sufficient, a suitable disinfectant must be employed for handwashing 4. Dust control - Hospital dust contains numerous bacteria and viruses, suppression of dust by wet dusting and vaccume cleaning are important control measures 5. Disinfection - The article used by the patient as well as patients urine, faeces, sputum should be properly disinfected proper sterilization of instrument should be enforced
  • 83. Dreamz Learning Innovations_____________________________________________ Page 83 6. Control of droplet infections-use of face masks, proper bed spacing, prevention of over crowding and ensuring adequate lighting and ventilation are important control measures 7. Nurshing technique - Barrier nurshing and task nurshing have been recommended to minimum cross infections 8. Administrative measures - There should be a hospital "control of infection committee" to formulate policies regarding control of hospital acquired infections
  • 84. Dreamz Learning Innovations_____________________________________________ Page 84 COHORT NURSHING - refer to the grouping of patients with a given infection within an isolated area short of strict (single room isolation) cohort grouping of infectious patients & nurshing them within an area of a hospital ward is widely recommended as a strategy for controlling transmission of health care acquired infections According to WHO highly aerosolized infected cases should be isolated & accommodated as follows in depending order or preference - Negative pressure room with the door closed** - Single room with their own bathroom facilities Cohart placement in an area with an independent air supply exhaust system & bathroom facilities The body exhaust system in total hip arthroplasty.
  • 85. Dreamz Learning Innovations_____________________________________________ Page 85 59. Most common nerve involvement in supracondylar fracture of humerus is A. Median nerve B. Ulnar nerve C. Posterior interosseus nerve D. Anterior interosseus nerve
  • 86. Dreamz Learning Innovations_____________________________________________ Page 86 59. Most common nerve involvement in supracondylar fracture of humerus is A. Median nerve B. Ulnar nerve C. Posterior interosseus nerve D. Anterior interosseus nerve A
  • 87. Dreamz Learning Innovations_____________________________________________ Page 87 SUPRACONDYLAR FRACTURE HUMERUS • Radial nerve is the most commonly injured nerve, the median nerves also sometimes affected • Median nerve may be injured but loss of function is usually temporary. • The nerve most commonly affected is median : radial palsies are rare, and the ulnar nerve is only involved when the fracture displacement is anterior (Watson & Jones) • In single choice best answer is - Median nerve (anterior interroseous branch)
  • 88. Dreamz Learning Innovations_____________________________________________ Page 88 Compression of median nerve within the carpal tunnel is the most common upper extremity compressive neuropathy Causes of carpal tunnel syndrome - Pregnancy amyloidosis, flexor tenosynovitis, over use phenomenon acute or chronic inflammatory conditions, traumatic disorders of the wrist, DM and hypothyroidism • Dunlop traction is used for supracondylar fracture of the humerus • Best prognosis after nerve repair is - Radial**
  • 89. Dreamz Learning Innovations_____________________________________________ Page 89 • Most common nerve used for monitoring during anaesthesia is ulnar nerve • Earliest reversal is orbicularis occuli supplied by facial nerve • Three bony point relationship is maintained in- supra condylar # humerus • Three bony point relationship is reversed m dislocation of elbow joint • Allen's test - is performed to identify patency of the radial and ulnar arteries at the wrist and is indicator of the integrity of the palmar arch, . • Neuronal degeneration is seen in crush nerve injury fetal development and senescence
  • 90. Dreamz Learning Innovations_____________________________________________ Page 90 Condition Features & Test Medial nerve palsy • Pointing index • Pen test (test Abductor pollicis brevis) • Benediction test • Oschner clasp test & opposition loss • Ape thumb deformity • Carpal tunnel syndrome Ulnar nerve • Book test (format sign) • Card test • Egawastest • Musician nerve Radial nerve • Wrist drop* • Thumb drop* • Finger drop* Sciatic nerve is the thickest* nerve of the body. It is the terminal branch of the lumbo-sacral plexus
  • 91. Dreamz Learning Innovations_____________________________________________ Page 91 60. Radio-isotopes used in all EXCEPT A. Fracture healing B. Infections of the joints C. Skeletal metastasis D. Bone tumors
  • 92. Dreamz Learning Innovations_____________________________________________ Page 92 60. Radio-isotopes used in all EXCEPT A. Fracture healing B. Infections of the joints C. Skeletal metastasis D. Bone tumors A
  • 93. Dreamz Learning Innovations_____________________________________________ Page 93 Clinical Applications of Radio-isotopes 1. Diagnosis of stress fracture (or other undisplaced fracture) 2. Detection of a small bone abscess or an osteoid osteoma 3. Investigation of loosening or infection around prosthesis 4. Diagnosis of femoral head ischaemia in perthe’s disease or avascular necrosis in adults 5. Early detection of bone metastases **the best indication for a bone scan is suspected multiple bony lesions such as in metastatic carcinomas, and lymphoma of bone • Bone- infarcts are seen in – Sickle cell anemia**
  • 94. Dreamz Learning Innovations_____________________________________________ Page 94 61. A lateral blow at the level of the knee joint may cause all except : A. Rupture of anterior cruciate ligament B. Rupture of medial collateral ligament C. Avulsion of lateral meniscus D. Bumper fracture of tibia
  • 95. Dreamz Learning Innovations_____________________________________________ Page 95 61. A lateral blow at the level of the knee joint may cause all except : A. Rupture of anterior cruciate ligament B. Rupture of medial collateral ligament C. Avulsion of lateral meniscus D. Bumper fracture of tibia C
  • 97. Dreamz Learning Innovations_____________________________________________ Page 97 62. Osteoporosis is a deficiency in: A. Calcium metabolism B. Calcium deposition C. Protein supporting tissue D. All of the above
  • 98. Dreamz Learning Innovations_____________________________________________ Page 98 62. Osteoporosis is a deficiency in: A. Calcium metabolism B. Calcium deposition C. Protein supporting tissue D. All of the above C
  • 99. Dreamz Learning Innovations_____________________________________________ Page 99 63. Achondroplasia is characterized by the following features except: A. Short extremities B. Normal trunk length C. Normal intelligence D. Normal ossification of cartilage
  • 100. Dreamz Learning Innovations_____________________________________________ Page 100 63. Achondroplasia is characterized by the following features except: A. Short extremities B. Normal trunk length C. Normal intelligence D. Normal ossification of cartilage D
  • 101. Dreamz Learning Innovations_____________________________________________ Page 101 64. Osteogenesis imperfecta is characterized by the following features except: A. Blue sclerae B. Brittle shell-like bones C. Multiple fractures D. Osteoporosis
  • 102. Dreamz Learning Innovations_____________________________________________ Page 102 64. Osteogenesis imperfecta is characterized by the following features except: A. Blue sclerae B. Brittle shell-like bones C. Multiple fractures D. Osteoporosis D
  • 103. Dreamz Learning Innovations_____________________________________________ Page 103 65. The treatment of osteoclastoma includes the following measures except: A. Curettage of tumor tissue & packing cavity with bone chips B. Excision with safety margin of bone C. Radiotherapy D. Chemotherapy
  • 104. Dreamz Learning Innovations_____________________________________________ Page 104 65. The treatment of osteoclastoma includes the following measures except: A. Curettage of tumor tissue & packing cavity with bone chips B. Excision with safety margin of bone C. Radiotherapy D. Chemotherapy D
  • 105. Dreamz Learning Innovations_____________________________________________ Page 105 66. The differential diagnosis of osteoid osteoma includes all the following except: A. Brodies' abscess B. Ossifying fibroma C. Ewing's tumor D. Metastatic thyroid nodule
  • 106. Dreamz Learning Innovations_____________________________________________ Page 106 66. The differential diagnosis of osteoid osteoma includes all the following except: A. Brodies' abscess B. Ossifying fibroma C. Ewing's tumor D. Metastatic thyroid nodule D
  • 107. Dreamz Learning Innovations_____________________________________________ Page 107 67. The management of Perthe's disease includes all except A. Broomstick plasters B. Compression nail plating C. Femoral osteotomy D. Innominate osteotomy
  • 108. Dreamz Learning Innovations_____________________________________________ Page 108 67. The management of Perthe's disease includes all except A. Broomstick plasters B. Compression nail plating C. Femoral osteotomy D. Innominate osteotomy B
  • 109. Dreamz Learning Innovations_____________________________________________ Page 109 68. Common complications of plastering of fractures is : A. Ischemia of the limb distal to the plaster. B. Venous congestion and venous thrombosis. C. Delayed or malunion of fracture. D. Orthostatic oedema of the limb:
  • 110. Dreamz Learning Innovations_____________________________________________ Page 110 68. Common complications of plastering of fractures is : A. Ischemia of the limb distal to the plaster. B. Venous congestion and venous thrombosis. C. Delayed or malunion of fracture. D. Orthostatic oedema of the limb: D
  • 111. Dreamz Learning Innovations_____________________________________________ Page 111 69. Treatment of Intracapsular fracture neck of femur in adult A. Skeletal traction B. POP Spica C. Excision of head of femur D. Closed reduction and internal fixation
  • 112. Dreamz Learning Innovations_____________________________________________ Page 112 69. Treatment of Intracapsular fracture neck of femur in adult A. Skeletal traction B. POP Spica C. Excision of head of femur D. Closed reduction and internal fixation C
  • 113. Dreamz Learning Innovations_____________________________________________ Page 113 • Treatments for intracapsular femoral neck fractures vary from internal fixation to hemiarthroplasty to total hip replacement
  • 114. Dreamz Learning Innovations_____________________________________________ Page 114 70. Common general complications of severe fracture pelvis include all except A. paralytic ileus. B. deep vein thrombosis of lower limbs. C. crush injury. D. pulmonary shock.
  • 115. Dreamz Learning Innovations_____________________________________________ Page 115 70. Common general complications of severe fracture pelvis include all except A. paralytic ileus. B. deep vein thrombosis of lower limbs. C. crush injury. D. pulmonary shock. C
  • 116. Dreamz Learning Innovations_____________________________________________ Page 116 71. Supracondylar fractures of the humerus in children A. Are common in children below the age of 10 B. More often result in a flexion injury than an extension injury C. May be complicated by a compartment syndrome in the upper arm, if the brachial artery is affected D. Feeble radial pulse is an absolute indication for immediate open exploration
  • 117. Dreamz Learning Innovations_____________________________________________ Page 117 71. Supracondylar fractures of the humerus in children A. Are common in children below the age of 10 B. More often result in a flexion injury than an extension injury C. May be complicated by a compartment syndrome in the upper arm, if the brachial artery is affected D. Feeble radial pulse is an absolute indication for immediate open exploration Answer A
  • 118. Dreamz Learning Innovations_____________________________________________ Page 118 These fractures are common in young children. Most are hypertension injuries. A brachial artery compromise may cause a forearm (not upper arm) compartment syndrome, and call for a trial of immediate closed reduction
  • 119. Dreamz Learning Innovations_____________________________________________ Page 119 72. Ulnar nerve compression: all of the following are true, EXCEPT: A. It may present with weakness rather than pain B. It usually requires nerve conduction studies for diagnosis of site compression C. Surgery is indicated for pain, but not for weakness D. it is less common than the carpal tunnel syndrome
  • 120. Dreamz Learning Innovations_____________________________________________ Page 120 72. Ulnar nerve compression: all of the following are true, EXCEPT: A. It may present with weakness rather than pain B. It usually requires nerve conduction studies for diagnosis of site compression C. Surgery is indicated for pain, but not for weakness D. it is less common than the carpal tunnel syndrome Answer C
  • 121. Dreamz Learning Innovations_____________________________________________ Page 121 Surgery should be done if weakness is present, even in the ab-, sence of pain.
  • 122. Dreamz Learning Innovations_____________________________________________ Page 122 73. A 10-year-old boy presents with a swelling in the upper humerus. Which of the following is the most cause? A. Osteosarcoma B. Metastatic tumor C. Myeloma D. Lymphoma
  • 123. Dreamz Learning Innovations_____________________________________________ Page 123 73. A 10-year-old boy presents with a swelling in the upper humerus. Which of the following is the most cause? A. Osteosarcoma B. Metastatic tumor C. Myeloma D. Lymphoma Answer A
  • 124. Dreamz Learning Innovations_____________________________________________ Page 124 Acute osteomyelitis, osteosarcomas and Ewing's tumors are among the commonest causes of swellings in the upper humerus in children.
  • 125. Dreamz Learning Innovations_____________________________________________ Page 125 74. Shoulder dislocation: the following are all true EXCEPT: A. It is the commonest of joint dislocations in adults B. It is predisposed to by a Hill-Sach's lesion C. It is usually posterior D. It can be reduced by the Kocher method
  • 126. Dreamz Learning Innovations_____________________________________________ Page 126 74. Shoulder dislocation: the following are all true EXCEPT: A. It is the commonest of joint dislocations in adults B. It is predisposed to by a Hill-Sach's lesion C. It is usually posterior D. It can be reduced by the Kocher method Answer C
  • 127. Dreamz Learning Innovations_____________________________________________ Page 127 Most shoulder dislocations are anterior.
  • 128. Dreamz Learning Innovations_____________________________________________ Page 128 75.Fractures of the proximal humerus A. Are increased in incidence in older persons B. Are more likely to need surgery in elderly patients C. Are classified according to mason's classification D. All of the above are true
  • 129. Dreamz Learning Innovations_____________________________________________ Page 129 75.Fractures of the proximal humerus A. Are increased in incidence in older persons B. Are more likely to need surgery in elderly patients C. Are classified according to mason's classification D. All of the above are true Answer A
  • 130. Dreamz Learning Innovations_____________________________________________ Page 130 Humeral fractures are commoner in older persons.
  • 131. Dreamz Learning Innovations_____________________________________________ Page 131 76.A 25 year old male presents to the emergency department after a road traffic accident. Clinical examination shows a blood pressure of 100/70 and mild abdominal tenderness. Iliac crest "springing" for a pelvic fracture is positive. Which of the following is the LEAST important next step in management? A. Start immediate intravenous fluids B. Conduct a careful examination for other injuries C. Carry out immediate diagnostic peritoneal lavage (DPL) for intra-abdominal trauma D. Send blood for hemoglobin levels and cross-matching
  • 132. Dreamz Learning Innovations_____________________________________________ Page 132 76.A 25 year old male presents to the emergency department after a road traffic accident. Clinical examination shows a blood pressure of 100/70 and mild abdominal tenderness. Iliac crest "springing" for a pelvic fracture is positive. Which of the following is the LEAST important next step in management? A. Start immediate intravenous fluids B. Conduct a careful examination for other injuries C. Carry out immediate diagnostic peritoneal lavage (DPL) for intra-abdominal trauma D. Send blood for hemoglobin levels and cross-matching Answer C
  • 133. Dreamz Learning Innovations_____________________________________________ Page 133 All patients with potentially significant trauma require at least one, and preferably two, i.v. lines for fluid infusion and blood. Every patient requires a quick clinical examination of the entire body. Except in the direst circumstances, every patient requires detailed clinical examination. Blood transfusion in this patient may be required, even if the hemoglobin is normal, but a DPL is not indicated at this stage, when the hemodynamic stability of the patient has not been confirmed.
  • 134. Dreamz Learning Innovations_____________________________________________ Page 134 77.A 55-year-old laborer presents to the casualty of a primary health center after a fall from a height. Plain X-rays confirm a fracture of both pubic rami. The patient is hemodynamically stable, but during observation of 5 hours it is noted that he does not pass urine. At that time abdominal examination shows that the bladder is full. If a urologist is unavailable, the most appropriate next step is: A. Administer intravenous carbachol to stimulate bladder contraction B. Administer sympathomimetics to decrease prostatic congestion C. Pass a urethral catheter D. Carry out a suprapubic catheterization ' "
  • 135. Dreamz Learning Innovations_____________________________________________ Page 135 77.A 55-year-old laborer presents to the casualty of a primary health center after a fall from a height. Plain X-rays confirm a fracture of both pubic rami. The patient is hemodynamically stable, but during observation of 5 hours it is noted that he does not pass urine. At that time abdominal examination shows that the bladder is full. If a urologist is unavailable, the most appropriate next step is: A. Administer intravenous carbachol to stimulate bladder contraction B. Administer sympathomimetics to decrease prostatic congestion C. Pass a urethral catheter D. Carry out a suprapubic catheterization ' " Answer D
  • 136. Dreamz Learning Innovations_____________________________________________ Page 136 The other three steps mentioned are all disastrous. Even in tertiary care centers, where retrograde urethrography facilities exist, suprapubic catheterization is never a wrong step in a patient with pelvic fracture and retention
  • 137. Dreamz Learning Innovations_____________________________________________ Page 137 78.A 62 year old woman presents with a history of fall while bathing. On examination she is hemodynamically stable, but is unable to move the left leg, which is lying in an attitude of external rotation. The following are all true EXCEPT: A. The patient is likely to have an intracapsular fractureof the femoral neck B. Plain X-rays are enough for diagnosis, and CT is rarely required C. Surgical reduction and fixation or replacement is always needed, and conservative treatment is contraindicated D. The commonest complication of this fracture is malunion
  • 138. Dreamz Learning Innovations_____________________________________________ Page 138 78.A 62 year old woman presents with a history of fall while bathing. On examination she is hemodynamically stable, but is unable to move the left leg, which is lying in an attitude of external rotation. The following are all true EXCEPT: A. The patient is likely to have an intracapsular fractureof the femoral neck B. Plain X-rays are enough for diagnosis, and CT is rarely required C. Surgical reduction and fixation or replacement is always needed, and conservative treatment is contraindicated D. The commonest complication of this fracture is malunion Answer D
  • 139. Dreamz Learning Innovations_____________________________________________ Page 139 The commonest complications of intracapsular fracture are nonunion and avascular necrosis. Unlike pelvic fractures, plain X-rays are usually enough to diagnose femoral fractures. If the X-rays are inconclusive despite compelling clinical signs, the patient should undergo repeat X-rays after 2-3 days, or MRI.
  • 140. Dreamz Learning Innovations_____________________________________________ Page 140 79.A 13 year old boy sustains a closed, stable fracture of the upper tibia. After reduction and application of a cast, he complains of excessive calf pain. The following step should be taken: A. X-rays to check for cast fragments V B. Elevate the limb 5-10° C. Elevate the limb by about 90° D. Remove the cast
  • 141. Dreamz Learning Innovations_____________________________________________ Page 141 79.A 13 year old boy sustains a closed, stable fracture of the upper tibia. After reduction and application of a cast, he complains of excessive calf pain. The following step should be taken: A. X-rays to check for cast fragments V B. Elevate the limb 5-10° C. Elevate the limb by about 90° D. Remove the cast Answer D
  • 142. Dreamz Learning Innovations_____________________________________________ Page 142 The boy has probably developed an impending compartment syndrome in the leg. This is caused by insufficient vascularity in the affected limb. The first step is to remove the cast and elevate the limb, and observe. If the pain continues the patient will require a fasciotomy.
  • 143. Dreamz Learning Innovations_____________________________________________ Page 143 80. The following is true of ankle fractures: A. The most frequent mechanism is pronation of the forefoot and internal rotation at the ankle B. Swelling and tenderness are typically absent or negligible C. Undisplaced fractures should be treated with a cast D. Post-traumatic arthritis is rare at the ankle
  • 144. Dreamz Learning Innovations_____________________________________________ Page 144 80. The following is true of ankle fractures: A. The most frequent mechanism is pronation of the forefoot and internal rotation at the ankle B. Swelling and tenderness are typically absent or negligible C. Undisplaced fractures should be treated with a cast D. Post-traumatic arthritis is rare at the ankle Answer C
  • 145. Dreamz Learning Innovations_____________________________________________ Page 145 Fractures at the ankle result from a twist. The injury develops during supination of the forefoot with external rotation at the ankle. Tenderness is marked, and swelling occurs. The commonest late complication is arthritis.

Editor's Notes

  1. 94
  2. 95
  3. 97
  4. 98
  5. 99
  6. 100
  7. 101
  8. 102
  9. 103
  10. 104
  11. 105
  12. 106
  13. 107
  14. 108
  15. 109
  16. 110
  17. 111
  18. 112
  19. 114
  20. 115
  21. 116
  22. 117
  23. 118
  24. 119
  25. 120
  26. 121
  27. 122
  28. 123
  29. 124
  30. 125
  31. 126
  32. 127
  33. 128
  34. 129
  35. 130
  36. 131
  37. 132
  38. 133
  39. 134
  40. 135
  41. 136
  42. 137
  43. 138
  44. 139
  45. 140
  46. 141
  47. 142
  48. 143
  49. 144
  50. 145